(Real Numbers) & (Polynomials) - Study Module

You might also like

Download as pdf or txt
Download as pdf or txt
You are on page 1of 77

1

CHAPTER
Real Numbers

School Level
Number System
You must have studied different types of numbers in your earlier classes such as natural numbers, whole numbers,
integers, rational, irrational and real numbers Let’s revise them.

Real

Natural
1, 2, 3... Whole Rational
0, 1, 2... .333..., 3
4
.77, –1.32
Integers ...–3, –2, –1, 0, 1, 2, 3...

Irrational
1.010010001...
5, 2, 10, π

‰ Natural numbers: Natural numbers are those which are used for counting and ordering i.e., N = {1, 2, 3, 4, ...}.
‰ Whole numbers: Whole numbers are collection of all natural numbers including zero i.e., W = {0, 1, 2, 3, 4, ...}.
‰ Integers: An integer is a number that can be written without fractional components. Integers consist of natural numbers,
their negatives and zero i.e., Z = {....–4, –3, –2, –1, 0, 1, 2, 3, 4,....}.
‰ Prime numbers: A number which can only be divided by either 1 or itself is called a prime number.

E.g: 2, 3, 5, 7, etc.
NOTE: (i) Each prime number have only two factors.
E.g: (a) 2 = 1 × 2 (b) 3 = 1 × 3
(ii) 2 is the only even prime number.
‰ Composite numbers: A number which has more than two factors is known as a composite number.

E.g: 4, 6, 8, 10 etc.
NOTE: 1 is neither a prime nor a composite number.
p
‰ Rational numbers: Rational numbers are those which can be expressed as , where p and q are integers and q ≠
q
0 & co-prime. Collection of all rational numbers are denoted by Q.
3 5
E.g: 1, 0, , . etc.
2 3
‰ Irrational numbers: All real numbers which are not rational numbers are termed as irrational numbers. A
non-terminating and non-repeating decimal is an example of irrational number. Collection of all irrational number are
denoted by P.
E.g: 2, π etc.
‰ Real numbers : The collection of all the rational and irrational numbers is called real numbers and it is denoted by R.

Fundamental Theorem of Arithmetic


The fundamental theorem of arithmetic (FTA) which is also known as unique prime factorisation theorem, gives relationship
between prime numbers and composite numbers. It states that:
Theorem 1: Every composite number can be expressed as a product of primes, and this factorisation is unique, apart
from the order in which the prime factors occurs.
E.g: 12600 = 23⋅32⋅52⋅7
Thus, the composite number 12600 is expressed as product of powers of primes in ascending order and this decomposition
is unique.
Corallary : The prime factorisation of a natural number is unique, except for the order of its factors.
NOTE: In the prime factorisation of a number we write the primes in ascending order.
i.e. If x = pa1 pb2 pc3 .... pan, where pi are prime then p1 < p2 < ... < pn, and a, b, c, d, & a are natural number’s.

Knowledge Hub
™ If p a prime number. If p divides a2, then p divides a, where a is a positive integer.
™ Prime factor of a prime number is the number itself.

Number of Zeros in a Number


The number of zeros in a number is the minimum power of 2 and 5 in the prime factorisation of that number. i.e.
Number of zeros in a number = Minimum (power of 2, power of 5).

H.C.F. and L.C.M. of Numbers


H.C.F (Highest common factor) : The H.C.F. of two or more numbers is the largest positive integer that divides each
of the integers.
L.C.M. (Least common multiple): The L.C.M. of two or more numbers is the smallest positive integer which is exactly
divisible by each of the given numbers.
To find HCF and LCM of given numbers using Prime Factorisation Method or Fundamental Theorem of Arithmetic, first
express each number in the form of product of prime factors, then
HCF = Product of the smallest powers of common factors.
LCM = Product of the greatest power of each prime factor involved in the numbers.

2 Class-X MATHEMATICS P
W
Important relations

‰ L.C.M × H.C.F = Product of two numbers.

‰ L.C.M of two or more prime numbers is equal to their product.

‰ H.C.F of two or more prime numbers is always 1.

‰ For three positive integers (a, b and c), HCF (a, b, c) × LCM (a, b, c) ≠ a × b × c, where a, b, c are positive integers.

‰ The largest number which will divide a, b and c leaving remainders x, y and z respectively is,

H.C.F. of (a – x), (b – y) and (c – z).


‰ The smallest number which when divided by a, b and c leaves the same remainder r in each case is

(L.C.M of a, b and c) + r.

EXAMPLE
1. Find the number of zeros in 38 × 2 × 25 × 39 × 50. As the given number have more than two factors,
Sol. The number can be rewritten in prime factorisation \ (7 × 11 × 15 + 15) is a composite number.
form as 4. By using prime factorization method, find the
2 × 2 × 2 × 3 × 5 × 5 × 5 × 5 × 13 × 19 L.C.M and H.C.F. of 1296 and 2520.
= 23 × 3 × 54 × 13 × 19 Sol. 1296 = 2 × 2 × 2 × 2 × 3 × 3 × 3 × 3 = 24 × 34
In prime factorisation, power of 2 = 3 & 2520 = 2 × 2 × 2 × 3 × 3 × 5 × 7 = 23 × 32 × 5 × 7
power of 5 = 4. L.C.M. (1296, 2520) = 24 × 34 × 5 × 7 = 45360
So, number of zeros in the number H.C.F. (1296, 2520) = 23 × 32 = 72
= minimum (power of 2, power of 5) 5. Find the greatest number which can divide
1251, 9377 and 15628 leaving 1, 2 and 3, as
= minimum (3, 4) = 3 remainders respectively.
\ No. of zeros = 3 Sol. Numbers are: (1251 – 1), (9377 – 2) and
2. On an evening walk, three people step off (15628 – 3) or 1250, 9375 and 15625.
together and their steps measure 40 cm, 42 cm Required number is the HCF of 1250, 9375 and
and 45 cm, respectively. How much minimum 15625.
distance each should cover so that each can Now, 1250 = 2 × 54
cover the same distance in complete steps? 9375 = 3 × 55
Sol. 40 = 23 × 51, 42 = 21 × 31 × 71 15625 = 56
45 = 32 × 51 HCF = 54 = 625
As three people step off together, minimum \ 625 is the largest number that divides 1251,
distance covered by each one of them 9377 and 1568 leaving remainders 1, 2 and 3
= LCM of 40, 42, 45 respectively.
6. Find the smallest number which when divided by
= 23 × 32 × 51 × 71 = 2520 cm.
16, 20 and 24 will leave a remainder 5 in each
3. 7 × 11 × 15 + 15 is a composite number. Explain. case.
Sol. 7 × 11 × 15 + 15 = (7 × 11 + 1) × 15 Sol. The required smallest number
= (77 + 1) × 15 = 78 × 15 = 2 × 3 × 13 × 15 = (L.C.M of 16, 20 and 24) + 5 = 240 + 5
= 2 × 3 × 13 × 15 = 245.

Real Numbers 3
Revisiting Irrational and Rational Numbers
Theorem 2: Let p be a prime number. If p divides m2, then p divides m, where m is a positive integer.

EXAMPLE

1. Prove that 2 is an irrational number. Again on squaring both sides, we get


a2 = 4c2
Sol. Proof: Let's assume, to the contrary, that 2 is
rational. ⇒ 2b2 = 4c2[ a2 = 2b2]
a ⇒ b2 = 2c2
Then 2 = where a and b are co-prime, integers
b This means that 2 divides b2 and therefore 2
& b ≠ 0. divides b (again using Theorem 2).
So, 2b = a. Therefore, a and b have at least 2 as a common factor.
On squaring both sides, we get But this contradicts the fact that a and b do not
a2 = 2b2 have a common factors other than 1.
\ 2 divides a2. This contradiction has arisen because of our wrong
Now, using Theorem 2, it follows that 2 divides a. assumption that 2 is rational.
So, we can write a = 2c for some integer c. So, we conclude that 2 is irrational.

Knowledge Hub

™ If pq is irrational, then p + q will definitely be irrational.


™ If p is a positive prime number, then p is an irrational number.
™ Decimal expansions of irrational numbers are non-terminating and non-reccuring.
™ Decimal expansions of rational numbers are either terminating or non-terminating and recurring.
™ Sum of a rational number and an irrational number always gives irrational number.
™ Sum of two irrational numbers does not always give an irrational number.
™ Product of a non-zero rational number with an irrational number always gives an irrational number.
™ Product of two irrational numbers does not always give an irrational number.

EXAMPLE

1. Prove that 2 + 3 is irrational. x2 − 5


As x, 5 and 2 are rational numbers, is a
Sol. Let us assume, to the contrary, that 2 + 3 is a rational number. 2
rational number equals to 'x' & x is a rational number. x2 − 5
Hence, 6= is a rational number, which is
⇒x= 2+ 3 2
On squaring both sides, we get in contradiction to the fact that 6 is an irrational
number.
x2 = 2 + 3 + 2 3 ⋅ 2 =+
5 2 6
Hence our assumption is wrong. So, we conclude
x2 − 5 that 2 + 3 is an irrational number.
⇒ x2 = 5+2 6 ⇒ 6 =
2

4 Class-X MATHEMATICS P
W
Summary

™ Fundamental Theorem of Arithmetic: Every composite number can be expressed as a product of primes factors
and this factorisation is unique, apart from the order in which the prime factors occur.
™ The number of zeros in a number is the minimum power of 2 and 5 in the prime factorisation of that number. i.e.
Number of zeros in a number = Minimum (power of 2, power of 5).
™ Theorem: Let p be a prime number. If p divides m2, then p divides m, where m is a positive integer.

NCERT Exercise

Product of LCM and HCF = 182 × 13 = 2366


Exercise-I
Clearly, LCM (26, 91) × HCF (26, 91)
1. Express each number as a product of its prime = Product of 26 and 91.
factors: (b) By prime factorisation, we get:
(a) 140 (b) 156 510 = 2 × 3 × 5 × 17
(c) 3825 (d) 5005 92 = 2 × 2 × 23
(e) 7429

\ HCF of 510 and 92 = 2
Sol. (a) 22 × 5 × 7 (b) 22 × 3 × 13 and LCM of 510 and 92
(c) 32 × 52 × 17 (d) 5 × 7 × 11 × 13 = 22 × 3 × 5 × 17 × 23 = 23460
(e) 17 × 19 × 23
Now, HCF × LCM = 2 × 23460 = 46920 …(i)
2. Find the LCM and HCF of the following pairs of Product of numbers = 510 × 92 = 46920 (ii)
integers and verify that LCM × HCF = product
From (i) and (ii), we get:
of the two numbers.
(a) 26 and 91 LCM × HCF = Product of numbers
(b) 510 and 92 Hence, verified.
(c) 336 and 54 (c) By prime factorisation, we get:
Sol. (a) 26 and 91 336 = 2 × 2 × 2 × 2 × 3 × 7
On expressing 26 and 91 as product of its prime 54 = 2 × 3 × 3 × 3
factors, we have
\ HCF of 336 and 54 = 2 × 3 = 6
26 = 2 × 13 and LCM of 336 and 54 = 24 × 33 × 7 = 3024
91 = 7 × 13 Now, LCM × HCF = 3024 × 6 = 18144 …(i)
Hence, LCM (26, 91) = 2 × 7 × 13 = 182 Product of numbers = 336 × 54 = 18144…(ii)
and HCF (26, 91) = 13 From (i) and (ii), we get:
Verification: LCM × HCF = Product of number
Product of 26 and 91 = 26 × 91 = 2366 Hence, verified.

Real Numbers 5
3. Find the LCM and HCF of the following integers Therefore, it is clear from above that, 6n is not
by applying the prime factorisation method. divisible by 5 for any natural number n and hence,
(a) 12, 15 and 21 [CBSE 2020] it proves that 6n can never end with the digit 0 for
(b) 17, 23 and 29 any natural number n.
(c) 8, 9 and 25 6. Explain why 7 × 11 × 13 + 13 and 7 × 6 × 5 × 4
× 3 × 2 × 1 + 5 are composite numbers.
Sol. (a) By prime factorisation, we get:
12 = 2 × 2 × 3 Sol. For 7 × 11 × 13 + 13
15 = 3 × 5 = 13(7 × 11 + 1) = 13 × 78
21 = 3 × 7 = 13 × 13 × 3 × 2
HCF of 12, 15 and 21 = 3 Hence it is composite number
and LCM = 2 × 2 × 3 × 5 × 7 = 420. For 7 × 6 × 5 × 4 × 3 × 2 × 1 + 5
(b) By prime factorisation, we get: Taking 5 as a common factor, we get,
17 = 17 × 1 = 5(7 × 6 × 4 × 3 × 2 × 1 + 1)
23 = 23 × 1 = 5(1008 + 1) = 5 × 1009
29 = 29 × 1 Hence, 7 × 6 × 5 × 4 × 3 × 2 × 1 + 5 is a composite
number.

\ HCF of 17, 23 and 29 = 1
7. There is a circular path around a sports field.
and LCM = 17 × 23 × 29 = 11339.
Monika takes 18 minutes to drive one round
(c) By prime factorisation, we get:
of the field, while Rohit takes 12 minutes for
8 = 2 × 2 × 2 × 1 the same. Suppose they both start at the same
9 = 3 × 3 × 1 point and at the same time, and go in the same
25 = 5 × 5 × 1 direction. After how many minutes will they meet

\ HCF of 8, 9 and 25 = 1 again at the starting point?

and LCM of 8, 9 and 25 = 23 × 32 × 52 = 1800. Sol. We need to find the number of minutes after which
they will meet again at the starting point. For this,
4. Given that HCF (306, 657) = 9, find
there will be a smallest number that is divisible by
LCM (306, 657).
both 18 and 12 and that will be the time when both
Sol. HCF (a, b) × LCM (a, b) = ab meet again at the starting point. To find this we have
ab to take LCM of both numbers.
⇒ LCM (a, b) =
HCF(a, b) Therefore, LCM (18, 12) = 2 × 3 × 3 × 2 = 36
\ LCM (306, 657) Hence, Monika and Rohit will meet again at the
306 × 657 starting point after 36 minutes.
= =34 × 657 =22338
9
5. Check whether 6n can end with the digit 0 for Exercise-2
any natural number n.
1. Prove that 5 is irrational.
Sol. For a number 6n to end with the digit zero (0), it
Sol. Let us assume to the contrary that 5 is a rational
must be divisible by 5, as we already know that any
number.
number having unit place as 0 or 5 is divisible by 5.
a
Prime factorization of 6n = (2 × 3)n i.e. 5= (where, a and b are co-prime, integers
b
As we can see that, the prime factorization of 6n and b ≠ 0)
doesn’t contain prime number 5. ⇒ b 5=a

6 Class-X MATHEMATICS P
W
On squaring both sides, we have 1 a
So, = , where a and b are co-prime,
2 b
(b 5 )
2
= a 2 ⇒ 5b2 = a2...(i)
integers and b ≠ 0.
As, a2 is divisible by 5, so a is also divisible by 5
b
(by theorem 2).
⇒ a 2 =⇒
b 2=
a
So, we can write a = 5c, for some integer c and on

Q b and a are integers.
substituting the value of a in equation (i), we get
b
5b2 = (5c)2 ⇒ b2 = 5c2 So, is a rational number.
a
As, b2 is divisible by 5, it means b is also divisible So, 2 is a rational number which contradicts
by 5 (by theorem 2). the fact that 2 is irrational.
Therefore, a and b have at least 5 as a common This contradiction occured due to we assumed
factor. But this contradicts the fact that a and b 1
that is rational.
are co-prime. This contradiction has arisen because 2
1
of our incorrect assumption that 5 is a rational Hence, we conclude that is irrational.
2
number.
(b) Let us assume that 7 5 is rational.
Hence, 5 is an irrational number.
\ There exists co-prime, integers a and b (b ≠
2. Prove that 3 + 2 5 is irrational. 0) such that:

Sol. Let us assume to the contrary, that 3 + 2 5 is rational. a a


7 5 =
⇒ 5=
b 7b
i.e., where a, b are integers, co-prime & b ≠ 0 such a
Since a and b are integers, we get is rational
that 7b
and so 5 is rational.
a a
3+ 2 5 = ⇒ 2 5= −3 But this contradicts the fact that 5 is irrational.
b b
1a  This contradiction has arisen because of our
⇒= 5  − 3
2 b  incorrect assumption that 7 5 is rational.
1a  Hence, we conclude that 7 5 is irrational.
Since, a and b are integers, thus,  − 3  is a
2 b 
(c) Let us assume that 6 + 2 is rational.
rational number and therefore, 5 should also be

\ There exists co-prime, integers a and b (b ≠ 0)
a rational number. such that
But this contradicts the fact that 5 is irrational. a a
6 + 2 = ⇒ 2 = –6
Hence, we conclude that 3 + 2 5 is irrational. b b
a
3. Prove that the following are irrationals Since a and b are integers, we get – 6 is
b
1 rational and so 2 is rational.
(a) (b) 7 5
2 But this contradicts the fact that 2 is
(c) 6 + 2 irrational.
This contradiction has arisen because of our
1
Sol. (a) Let us assume to the contrary that is a incorrect assumption that 6 + 2 is rational.
2
rational number. Hence, we conclude that 6 + 2 is irrational.

Real Numbers 7
2. Match the following columns:
Fill in the Blanks
Column-I Column-II
1. 144 as a product of its prime factors is _______
P. Remainder when 667 is (i) 448
2. 5 − 3 is an _______ Number. divided by 22
3. The greatest possible number with which when we Q. Remainder when 448 is (ii) Yes
divide 37 and 58, leaving the respective remainder divided by 449
of 2 and 3, is _______
R. Remainder can be zero? (iii) No
4. The product of two numbers is equal to the _______
S. Divisor can be zero? (iv) 7
of their HCF and LCM.
5. The two numbers are 396 and 576 and their LCM is (a) P-(iv), Q-(iii), R-(ii), S-(i)
6336. The HCF of the numbers is _______ (b) P-(i), Q-(ii), R-(iii), S-(iv)
(c) P-(ii), Q-(iv), R-(i), S-(iii)
True and False Statements (d) P-(iv), Q-(i), R-(ii), S-(iii)

1. LCM of 6, 72 and 120 is 720.


2. 8n may end with the digit 0 for any natural number n.
Assertion & Reason Type Questions
3. If 19 divides a3 (where a is a positive integer), then Direction (Q. 1-3): In the following questions, a statement
19 divides a. of Assertion (A) is followed by a statement of Reason (R).
4. The quotient of two integers is always a rational Mark the correct choice as:
number. (a) Both Assertion (A) and Reason (R) are true
5. Product of 2 irrational numbers may or may not be and Reason (R) is the correct explanation of
an irrational number. Assertion (A).
(b) Both Assertion (A) and Reason (R) are true
Match the Following but Reason (R) is not the correct explanation
of Assertion (A).
1. Match the following columns :
(c) Assertion (A) is true but Reason (R) is false.
Column-I Column-II
(d) Assertion (A) is false but Reason (R) is true
P. H.C.F. of 306 and 657 (i) 2
1. Assertion (A): If H.C.F. of two numbers is 16 and
Q. H.C.F. of the smallest (ii) 5
their product is 3072, then their L.C.M. is 162.
composite number
and the smallest prime Reason (R): If p, q are two positive integers, then
number H.C.F. (p, q) × L.C.M. (p, q) = p × q.
R. H.C.F. of 475 and 495 (iii) 9 2. Assertion (A): 2 is an example of a rational number.
Reason (R): The square roots of all positive integers
S. Power of 2 in 144 (iv) 4 are irrational numbers.
(a) P-(i), Q-(iv), R-(iii), S-(ii) 3. Assertion (A): For any two positive integers p and
(b) P-(iii), Q-(ii), R-(i), S-(iv) q, HCF (p, q) × LCM (p, q) = p × q
(c) P-(iii), Q-(i), R-(ii), S-(iv) Reason (R): If the HCF of two numbers is 5 and
(d) P-(iv), Q-(i), R-(iii), S-(iv) their product is 150, then their LCM is 40.

8 Class-X MATHEMATICS P
W
1. The LCM of two numbers is 2400. Which of (a) Only Statement-I is true
the following can not be their HCF? (b) Both Statement-I & Statement-II is true
 [CBSE 2022] (c) Only Statement-II is true
(a) 300 (b) 400
(d) Neither Statement-I or Statement-II is true
(c) 500 (d) 600
7. HCF of two numbers is 27 and their LCM is 162.
2. The number of zeroes in number n, if n = 23 × 32
If one of the numbers is 54, then other number is
× 52 × 7, is equal to
 [CBSE 2020]
(a) 2 (b) 1
(a) 36 (b) 35
(c) 5 (d) 6
(c) 9 (d) 81
3. The least number that is divisible by all the natural
8. If a = 2 3 × 3, b = 2 × 3 × 5, c = 3 p × 5 and
numbers from 1 to 10 (both inclusive) is
[Exemplar] LCM (a, b, c) = 23 × 32 × 5, then p =
(a) 10 (b) 100 (a) 6 (b) 4
(c) 504 (d) 2520 (c) 9 (d) 2
4. If two positive integers a and b can be expressed as 9. What is the greatest possible speed at which a girl
p = ab2 and q = a3b; a, b being prime numbers, then can walk 95 m and 171 m in an exact number of
LCM (p, q) is [Exemplar] minutes? [CBSE 2022]
(a) ab 2
(b) a b 2 (a) 17 m/min. (b) 19 m/min.
3
(c) a b 2 (d) a3b3 (c) 23 m/min. (d) 13 m/min.
5. Find the greatest number which when divides 70 10. Find the smallest number which is a perfect square
and 125, leaves remainder 5 and 8 respectively. and is divisible by each of 16, 20 and 24.
(a) 875 (b) 10 (a) 600 (b) 2800
(c) 13 (d) 1680 (c) 3600 (d) 2400
6. Statement-I: It two positive integers a and b are
written as a = x3y2 and b = xy3; x, y are prime 11. The L.C.M. of a and 18 is 36. The H.C.F. of a and
numbers, then HCF of a & b is x3y2. 18 is 2. Find the value of number a.
Statement-II: If n is a natural number then 12n can (a) 4 (b) 5
not end with digit 5. (c) 6 (d) 1

2 pa
Very Short Answer Type Questions
2 qb
1. If HCF (336, 54) = 6, find LCM (336, 54)
2 br
 [CBSE 2019]
2. Find the prime factors of 1080. 17
3. Numbers p, q and r are missing in the following 4. Find the total number of factors of prime numbers.
factorisation. Find them.  [CBSE 2020]

Real Numbers 9
5. There is a number 6n, where n is a natural number.
Check if there is any value of n ∈ N exists, for Long Answer Type Questions
which 6n is divisible by 7.
1. If a, b, c, d be positive rationals such that
6. Find a number which is greater than 3 and when it gets
divided by 4, 5 and 6 always leaves 3 as remainder. a + b =+ c d , then prove that either a = c and
7. Express 429 as a product of its prime factors. b = d or b and d are squares of rationals.
[CBSE 2019] 2. Prove that 7 + 11 is an irrational number.
8. For two numbers 150 and 100, it is given that
L.C.M. (150, 100) = 300, find H.C.F. (150, 100).
Case Study Type Questions
9. Find a rational number between 2 and 3.
 [CBSE 2019] Case Study-I
The department of Mathematics is conducting an
Short Answer Type Questions International Seminar. In the seminar, the number of
participants in Mathematics, Physics and Computer
1. Find the number which is nearest to 100000 and
Science are 60, 84 and 108 respectively. The arrangement
greater than 100000 which is exactly divisible by
was made such that, the same number of participants, all
each of 8, 15 and 21.
of them being in the same subject, are to be seated in each
2. In a school 437 girls and 342 boys have been room. A separate room was alloted for all the official other
divided into groups, so that each group has the same than participants.
number of students and no group has boys and girls
mixed. What is the least number of groups needed? 1. Total number of participants will be
(a) 160 (b) 84
3. Pooja multiplied a number 484 with a certain number
to obtain the result 3823a. Find the value of a. (c) 220 (d) none of these

4. If first 100 multiples of 10 are multiplied with each 2. Find the LCM of 60, 84 and 108.
other, then find the number of zeroes at the end of (a) 3780 (b) 840
the product. (c) 544320 (d) 12
5. Three people running around a rectangular track, can 3. If in each room, the same number of participants
complete one turn in 2 hours, 4 hours and 5.5 hours are to be seated and all of them being in the same
respectively. When will they meet at the starting point? subject, then find the least number of rooms
6. 4 bells ring together at 9.00 a.m. They ring after required for them.
7 seconds, 8 seconds, 11 seconds and 12 seconds (a) 16 (b) 20
respectively. How many times will they ring together (c) 14 (d) 21
again in the next 3 hours?
7. Find HCF and LCM of 404 and 96 and verify that Case Study-II
HCF × LCM = Product of the two given numbers. Birbal, Akbar and Mohit are playing a game. Birbal climbs
 [CBSE 2018] 5 stairs and gets down 2 stairs in one turn. Akbar goes up
2+ 3 by 7 stairs and comes down by 2 stairs every time. Mohit
8. Prove that is an irrational number, given 5
5 goes 10 stairs up and 3 stairs down each time.
that 3 is an irrational number. [CBSE 2019] Doing this they have to reach the nearest point of the 100th
9. Prove that (3 − 5) is an irrational number. stairs and they will stop once they find it impossible to go
10. Prove that 3 is an irrational number. forward. They can not cross the 100th stair in any way.
[CBSE 2020] 1. Who reaches the nearest point?
11. Prove that p + q is irrational, where p, q are (a) Birbal (b) Akbar
primes. [Exemplar] (c) Mohit (d) All together

10 Class-X MATHEMATICS P
W
2. How many times can they meet in between on the (b) Akbar and Mohit will meet for the first time on
same stair? the 35th stair.
(a) 3 (b) 4 (c) Birbal and Mohit will meet for the first time on
(c) 5 (d) 0 the 21st stair.
3. Who takes the least number of steps to reach near a (d) Birbal and Akbar will meet for the first time on
hundred? 21st stair.
(a) Birbal (b) Akbar 5. What is the second stair where any two out of three
(c) Mohit (d) All will meet together?
4. What is the first stair where any two out of three (a) Birbal and Akbar will meet on 21st stair.
will meet together? (b) Akbar and Mohit will meet on the 35th stair.
(a) Birbal and Akbar will meet for the first time on (c) Birbal and Mohit will meet on the 21st stair.
the 15th stair. (d) Birbal and Mohit will meet on the 35th stair.

ANSWER KEY

Quick Recall
Fill in the Blanks
1. 24 × 32 2. Irrational Number 3. The number will be HCF(35,55) = 5 4. Product
Product of the numbers (396 × 576)
5. HCF = = = 36
L.C.M. 6336
True and False Statements
1. False 2. False 3. True 4. False 5. True

Match the Following


1. (c) 2. (d)

Assertion & Reason Type Questions


1. (d) 2. (c) 3. (c)

Multiple Choice Questions


1. (c) 2. (a) 3. (d) 4. (c) 5. (c) 6. (c) 7. (d) 8. (d) 9. (b) 10. (c)
11. (a)

Subjective Questions
Case Study Type Questions
Case Study-I
1. (d) 2. (a) 3. (d)
Case Study-II
1. (a) 2. (d) 3. (c) 4. (a) 5. (c)

Real Numbers 11
Competitive Level

Euclid’s Division Lemma*


Theorem 3: Given two positive integers a and b, there exist unique integers q and r satisfying a = bq + r, 0 ≤ r < b.
Here, when a is divided by b, i.e a is dividend, b is the divisor, q and r are obtained as quotient and remainder respectively.
Let’s have a look at some examples.
(i) Consider number a = 27 and b = 5, 27 = 5 × 5 + 2
Comparing this with a = bq + r, we get

q = 5, r = 2 and 0 ≤ r < b (as 0 ≤ 2 < 5).
(ii) Consider positive integers a = 4 and b = 14.
4 = 14 × 0 + 4
Comparing this with a = bq + r, we get q = 0, r = 4 and 0 ≤ r < b (as 0 ≤ 4 < 14).
(iii) Consider positive integers a = 18 and b = 6
18 = 6 × 3 + 0
Comparing this with a = bq + r, we get q = 3, r = 0 and 0 ≤ r < b as 0 ≤ 0 < 6
NOTE: q and r can be zero.
The relation a = bq + r, 0 ≤ r < b is just a restatement of long division process of number 'a' by a number 'b' in which
'q' is obtained as quotient and 'r' is obtained as remainder.
Thus, dividend = divisor × quotient + remainder ⇒ a = bq + r.

Knowledge Hub
™ Lemma is a proven statement used for proving an another statement.
™ Euclid's Division Lemma hold not only for positive integers but for all integers a & b, b ≠ 0.

EXAMPLE
1. Prove that the square of any positive integer = 5(5m2)
cannot be of the form 5q + 2 or 5q + 3 for any = 5q  [Let q = 5m2, q is an integer]
integer q. [Exemplar, CBSE 2015, 2020]
Case-II: (5m + 1)2 = 25m2 + 10m + 1
Sol. We know that any positive integer can be of the
= 5(5m2 + 2m) + 1
form 5m, 5m + 1, 5m + 2, 5m + 3 or 5m + 4, for
some integer m. = 5q + 1 [Let q = 5m2 + 2m, q is an integer]

Here 0 ≤ r < 5, ⸪ 0 ≤ r < b Case-III: (5m + 2)­2 = 25m2 + 20m + 4


Therefore, we have = 5(5m2 + 4m) + 4
Case-I: (5m)2 = 25m2 = 5q + 4 [Let q = 5m2 + 4m, q is an integer]

12 Class-X MATHEMATICS P
W
Case-IV: (5m + 3)2 = 25m2 + 30m + 9 = 4m2 + 1 + 4m + 4n2 + 1 + 4n
= 25m2 + 30m + 5 + 4
On rearranging, we get
= 5(5m2 + 6m + 1) + 4
= 5q + 4 [Let q = 5m2 + 6m + 1, q is an integer] = 4m2 + 4n2 + 4m + 4n + 1 + 1
Case-V: (5m + 4)2 = 25m2 + 40m + 16 = 4(m2 + n2) + 4(m + n) + 2
= 25m2 + 40m + 15 + 1
= 2[2(m2 + n2) + 2(m + n) + 1]
= 5(5m2 + 8m + 3) + 1
= 5q + 1 [Let q = 5m2 + 8m + 3, q is an integer] = 2q [Let q = 2(m2 + n2) + 2(m + n) + 1]
This proves that for any integer q, the square
As x2 + y2 is of the form 2q, this means that it is
of any positive integer cannot be of the form
5q + 2 or 5q + 3. divisible by 2 and hence it is even.
2. If x and y are both odd positive integers, then
Again x2 + y2 = 4[(m2 + n2) + (m + n)} + 2
prove that x2 + y2 is even but not divisible
by 4. [Exemplar] = 4q + 2 [Let q = (m2 + n2) + (m + n)]
Sol. For some integers m and n,
⇒ x2 + y2 is even and leaves remainder 2 on
Let x = 2m + 1 and y = 2n + 1
dividing by 4.
We have x2 + y2
\ x2 + y2 = (2m + 1)2 + (2n + 1)2 Therefore, x2 + y2 is not divisible by 4.

Euclid's Division Algorithm


Euclid’s Algorithm is a method to compute highest common factor (HCF) or the greatest common divisor (GCD) of two
given positive integers a & b.
So, let's understand Euclid's division algorithm more clearly.
For obtaining HCF of two positive integers, say a and b, with a > b, follow the steps given below:
Step-1: Apply Euclid's division lemma to a and b, so that we can find whole numbers q and r, such that a = bq + r (0 ≤ r < b).
Step-2: If r = 0, b is the HCF of a and b. If r ≠ 0, apply the division lemma to b and r.
Step-3: The process is continued till we get the remainder zero. The divisor we get at this stage will be the required HCF.
NOTE: Euclid’s division lemma and algorithm are so closely interlinked that often lemma is called as the division
algorithm.

Knowledge Hub
™ An algorithm is a series of well defined steps which gives a procedure for solving a type of problem.
™ The basic principle behind this algorithm is the fact that,
HCF (a, b) = HCF (b, r)
where, a, b, & r are dividend, divisor & remainder respectively.

Real Numbers 13
EXAMPLE
1. Using Euclid’s algorithm find the HCF of 25152 = HCF (148, 50) = HCF (346, 148) = HCF (840,
and 2026. 346) = HCF (2026, 840) = HCF (25152, 2026).
Sol. Two given numbers are 25152 and 2026. Now, 2. A trader has 1232 Dettol soaps and 64 Pears
we start dividing larger number by the smaller soaps. He packs them in such a way that each
number. box has only one type of soap. If each box has
Step-1: Since, 25152 > 2026, thus we apply the the same quantity of the soaps, calculate the
division algorithm to 25152 and 2026. number of soaps in each box so that the total
number of boxes is the lowest.
25152 = 2026 × 12 + 840
Sol. To find the number of soaps in each box such
Step-2: Since, r = 840 ≠ 0, thus we apply division
that the number of boxes is the minimum, we find
algorithm to 2026 and 840.
HCF (1232, 64).
2026 = 840 × 2 + 346
Let's find their HCF using Euclid's algorithm.
Step-3: Since, r = 346 ≠ 0, thus we apply division
We have, 1232 = 64 × 19 + 16
algorithm to 840 and 346.
Again, 64 = 16 × 4 + 0
840 = 346 × 2 + 148
So, the HCF of 1232 and 64 is 16.
Step-4: Since, the r = 148 ≠ 0, thus we apply
division algorithm to 346 and 148. Therefore, the number of soaps in each box such
that the number of boxes is the least is 16.
346 = 148 × 2 + 50
3. Find the HCF of 420, 130 and 600, by using
Step-5: Since, r = 50 ≠ 0, thus we apply division
Euclid's division algorithm.
algorithm to 148 and 50.
Sol. To find the HCF of 420, 130 & 600, we will first
148 = 50 × 2 + 48
find the HCF of 420 & 130 by Euclid’s division
Step-6: Since, r = 48 ≠ 0, thus we apply division
algorithm.
algorithm to 50 and 48.
By using division algorithm, we get
50 = 48 × 1 + 2
420 = 130 × 3 + 30 (r = 30 ≠ 0)
Step-7: Since, r = 2 ≠ 0, thus we apply division
130 = 30 × 4 + 10 (r = 10 ≠ 0)
algorithm to 48 and 2.
30 = 10 × 3 + 0 (r = 0)
48 = 2 × 24 + 0
So, HCF (420, 130) = 10
The remainder, r has now become zero, so here
we stop our procedure. Since the divisor in this Now, we will find the HCF of 10 and 600
step is 2, thus the HCF of 25152 and 2026 is 2. 600 = 10 × 60 + 0 (r = 0)
Observing the above solution, we can say that \ HCF (10, 600) = 10
2 = HCF (48, 2) = HCF (50, 48)
Hence, HCF (420, 130, 600) = 10.

p
Theorem 4: Let x is a rational number whose decimal expansion terminates, then x can be expressed as q , where p and

q are co-primes, and the prime factorisation of q is of the form 2m5n, where n, m are non-negative integers.
875 7 × 53 7
E.g: 0.0875 =
= 4 =
4 4 4
10 2 ×5 2 ×5
Here, q = 24 × 5 which is of the form 2m × 5n, m, n ∈ W

14 Class-X MATHEMATICS P
W
a
Theorem 5: Let x = be a rational number in which the prime factorization of b is of the form 2m 5n, where m, n
b
are non-negative integers, then x has a decimal expansion which terminates.

64 26 29 512 9 9 × 22 36 36
E.g: (i) = = = = 0.512 (ii)
= =2 2
= 2 2
= 0.36
125 5 3
(2 × 5) 3
10 3
25 5 × 2 (2 × 5) (10)
a
So, a rational number of the form , where b is of the form 2m5n can be converted to an equivalent rational number of
b
the form p where q is a power of 10. Therefore, the decimal expansion of such rational numbers terminate.
q
a
Theorem 6: Let x = be a rational number in which the prime factorization of b is not of the form 2m5n, where m, n
b
are non-negative integers, then x has a decimal expansion which is recurring i.e. non-terminating repeating.
1
E.g: = 0.142857142857...
7
As, denominator 7 is not of the form 2m5n, therefore this rational number will not terminate.
It can be concluded from the above discussion that the decimal expansion of every rational number is either terminating
or non-terminating repeating.

Relation Between HCF & LCM of Three Numbers


a ⋅ b ⋅ c ⋅ HCF (a, b, c)
(i) LCM (a, b, c) =
HCF (a, b) ⋅ HCF (b, c) ⋅ HCF (a, c)
a ⋅ b ⋅ c ⋅ LCM (a, b, c)
(ii) HCF (a, b, c) =
LCM (a, b) ⋅ LCM (b, c) ⋅ LCM (a, c)

HCF & LCM of Fractions


p a
For fractions and
q b
L.C.M of numerators L.C.M.(p, a )
(i) L.C.M of fraction = =
H.C.F of denominators H.C.F. (q, b)

H.C.F of numerators H.C.F. (p, a )


(ii) H.C.F of fraction = =
L.C.M of denominators L.C.M.(q, b)

20 52
E.g: Find the LCM and HCF of numbers & .
27 87
Sol. 20 = 22 × 5; 27 = 33; 52 = 22 × 13; 87 = 3 × 29

20 52 LCM(20,52) 22 × 5 × 13 260
\ LCM of = and = =
27 87 HCF(27,87) 3 3

20 52 HCF(20,52) 22 4
HCF of and
= = = 3
27 87 LCF(27,87) 3 × 29 783

Real Numbers 15
H.C.F and L.C.M. of Decimals
Follow the following procedure to find LCM & HCF
Step-1: Convert each of the decimal numbers to numbers with same decimal places.
Step-2: Remove the decimal point and find the HCF and LCM.
Step-3: In the resultant HCF or LCM put the decimal point as there was in numbers after step-1.
E.g. Find the LCM and HCF of 0.34 and 2.7.
Sol. 0.34 and 2.7 can be rewritten as 0.34 and 2.70.
Now, removing the decimal point we get-
34 and 270
HCF (34, 270) = 2
LCM (34, 270) = 270 × 17 = 4590
Putting the decimal point after two places as was earlier.
HCF (0.34, 2.7) = 0.02
LCM (0.34, 2.7) = 45.9

Important Concepts Related to H.C.F & L.C.M of Numbers


U Total number of numbers (upto a certain number a) which are divisible by a certain integer b is the quotient q,
when a is divided by b.
E.g. Find how many numbers up to 594 are divisible by 15?
Sol. We divide 594 by 15.
594 = 39 × 15 + 9
The quotient obtained is the required number. Therefore, there are 39 such numbers upto 594 which are divisible
by 15.
E.g. Find how many numbers up to 600 are divisible by 7 and 5 together?
Sol. As, L.C.M. of 7 and 5 = 35
\ We divide 600 by 35
600 = 17 × 35 + 5
Therefore, there are 17 such numbers upto 600 which are divisible by 7 and 5 together.
� Two numbers when get divided by a certain divisor leave remainders r1 and r2 and when their sum is divided by
the same divisor, gives remainder r3. Then, the divisor is given by (r1 + r2 – r3).
E.g. Two numbers when divided by a certain divisor leave remainders 473 and 298, respectively. When their sum
is divided by the same divisor, the remainder is 236. Find the divisor.
Sol. The required divisor
= 437 + 298 – 236 = 499
Hence, the required divisor is 499.
U The smallest number which when divided by a, b and c leave the remainders x, y and z respectively. Such that
(a – x) = (b – y) = (c – z) = k(say) is (L.C.M. of a, b and c) – k.

16 Class-X MATHEMATICS P
W
E.g. Find the smallest number which when divided by 12, 15 and 21 leave the remainders 7, 10 and 16 respectively.
Sol. Since (12 – 7) = (15 – 10) = (21 – 16) = 5, therefore the required smallest number,
= (L.C.M of 12, 15 and 21) – 5 = 420 – 5 = 415.
U The largest number which will divide a, b and c leaving the same remainder in each case.
(a) When the value of remainder is not given:
Required number = H.C.F. of |(a – b)|, |(b – c)| and |(c – a)|
(b) When the value of remainder r is given:
Required number = H.C.F. of (a – r), (b – r) and (c – r).
E.g. Find the largest number which on dividing 34, 90 and 104 leaves the same remainder in each case.
Sol. The required largest number.
= H.C.F. of |(a – b)|, |(b – c) and (c – a)|
= H.C.F. of |(34 – 90)|, |(90 – 104)| and |(104 – 34)| = H.C.F. of 56, 14 and 70 = 14
E.g. Find the largest number which will divide 155 and 192 and leaves the remainder 7 in each case.
Sol. The required largest number
= H.C.F. of (155 – 7) and (192 – 7) = H.C.F. of 148 and 185 = 37

Number of Factors of a Given Number


Let N be a composite number, and its prime factors be a, b, c, d,....... etc. and k, l, m, n.... etc. be the powers of
a, b, c, d.... etc. respectively i.e., N can be expressed as N = ak ⋅ bl ⋅ cm ⋅ d n....

Then, the number of total factors or divisors of N is (k + 1) (l + 1) (m + 1) (n + 1)....


E.g: Find the total number of factors of 1080
Sol. 1080 = 2 × 2 × 2 × 3 × 3 × 3 × 5
540 = 23 × 33 × 51
Therefore total number of factors of 1080 is (3 + 1) (3 + 1) (1 + 1) = 32

Number of Odd Factors


To find the number of odd factors of a number N, firstly express the number N as
N = (a1p × a2q × a3r × ....) × (ex)
(where, a1, a2, a3, ... are the odd prime factors and e is the even prime factor)
Then the total number of odd factors = (p + 1)(q + 1)(r + 1)
E.g: The number of odd factors of 72 is ....
Sol. 72 = 23 × 32
\ Number of odd factors = (2 + 1) = 3

Number of Even Factors


Number of even factors of a number = (Total number of factors of the given number) – (Total number of odd factors of
the given number).

Real Numbers 17
E.g: Find the number of even factors (or divisors) of 72
Sol. Total number of factors = (3 + 1) (2 + 1) = 12
Number of odd factors = (2 + 1) = 3
Number of even factors = Total number of factors – No. of odd factors = 12 – 3 = 9

Sum of Factors of a Given Number


Let N be a composite number and its prime factors be a, b, c, d.... etc. and k, l, m, n.... etc. be the powers of a, b, c, d....
etc. respectively i.e., N can be expressed as N = ak . bl . cm . dn....
(a k +1 − 1)(bl +1 − 1)(c m +1 − 1)(d n +1 − 1)...
then the sum of all the factors or divisors of N =
(a − 1)(b − 1)(c − 1)(d − 1)...
E.g: Find the sum of the factors of 180.
Sol. 180 = 22 × 32 × 5
(22+1 − 1)(32+1 − 1)(51+1 − 1) 7 × 26 × 24
\ Sum of the factors of 180 = = = 546
(2 − 1)(3 − 1)(5 − 1) 1× 2 × 4

Product of Factors
n
The product of factors of a composite number N is given by N 2 , where n is the total number of factors of the composite
number N.
E.g: N = 24 = 23 × 3
No. of factor is (3 + 1) (1 + 1) = 8
8
Product of factors of =
N 24
= 2 244

Irrational Numbers in Ascending and Descending Orders


Irrational numbers with same index can be compared. To compare irrational numbers with different indexes, we have to
convert them into same index and then compare.
Writing irrational numbers in ascending and descending order:
(i) To write irrational numbers (with same index) from smaller to greater is known as ‘ascending order’.
(ii) To write irrational numbers (with same index) from greater to smaller is known as ‘descending order’.
Converting irrational numbers with different indexes to irrational numbers with same index
(i) Write the orders of irrational numbers given.
(ii) Find the least common multiple of indexes of irrational numbers.
(iii) Make the indexes same with the help of least common multiple found.
(iv) Compare the radicands.
E.g: Write 3
24 , 3 56 , 3 29 in ascending order.
Sol. Q Radicals of all the given numbers are same and ascending order of radicands 24, 56 & 29 is 24, 29 & 56.
So, 3
24 < 3 29 < 3 56

18 Class-X MATHEMATICS P
W
E.g: Write 2
3 , 3 4 , 4 2 in desending order.
Sol. LCM of the indexes 2, 3 & 4 of the given numbers is 12.
1 6 1
Now, 2
3 3=
= 2
312= (36 )12= 12 6
3= 12
729
1 4 1
12
3
= = (44 )12
4 4= 412 3
= 4
4= 12
256
1 3 1
3 2 12 3
4
2 2= 2 = (2 )=
= 4 12
2= 12
8
Thus, all the number are in same index & 729 > 256 > 8.
So, 12
729 > 12 256 > 12 8 i.e. 2
3>34>42

Unit Digit of a Number Raised any Power


U Use the following table to find the unit digit of a number, ending with 0, 1, 5 or 6, raised to any power

Any Power of Unit digit


0 0
1 1
5 5
6 6
E.g: (i) The unit digit of (30)35 = 0. (ii) The unit digit of (71)13 = 1.
(iii) The unit digit of (35)12 = 5 (iv) The unit digit of (26)23 = 6.
U The unit digit of a number, ending with 4 or 9 and raised to a power is given by-

Number Unit digit when power is odd Unit digit when power is even
4 4 6
9 9 1
E.g: (i) The unit digit of (74)53 = 4 (ii) The unit digit of (74)24 = 6
(iii) The unit digit of (69)24 = 1
(iv) The unit digit of 6933 = 9
U The unit digit of the power of 2, 3, 7 and 8 repeat after every cycle of 4 steps therefore to find the unit digit of any
power of 2, 3, 7 and 8, first divide the power by 4 and look at the remainder, (0, 1, 2 or 3) and use the following table:
Remainder
Unit digit of 0 1 2 3
2 6 2 4 8
3 1 3 9 7
7 1 7 9 3
8 6 8 4 2

E.g: (i) The unit digit of 8725


Q remainder when 25 is divided by 4 is 1.
So, unit digit of 8725 = 7
(ii) The unit digit of 7232.

Q remainder when 32 is divided by 4 is 0.
So, unit digit of 7232 = 6.

Real Numbers 19
Last Two Digits of a Product
If X × Y is the product where X =….ab and Y =….cd then the last two digits of the product can be found with the following
process:
Step-1: Unit digit of X × Y is the unit digit in the product of b and d. If any, the excess digit will be carried forward to next step.
Step-2: Add the product of a and d, and c and b i.e. find a × d + c × b.
Step-3: The unit’s digit in above step becomes ten’s digit of the number. If there was any excess digit in Step 1 then it will
be added to a × d + c × b and the unit’s digit in the resultant will be ten's digit of the product.
E.g: X = 5647 and Y = 6384
Step-1: 7 × 4 = 28. Unit digit of the product XY = 8 and 2 is excess digit.
Step-2: a × d + c × b = 4 × 4 + 8 × 7 = 72
Step-3: Add 2 (excess digit) to 72 = 74.
Ten’s digit of XY = 4.
So, last two digit of XY = 5647 × 6384 is 48 i.e. 4 and 8.

Last Two Digit of a Number Raised to Some Power


Let the number is XY.
Case-1: If unit digit of X is 1
Unit digit of X Y will be 1.
Ten’s digit is the unit digit in the product of ten’s digit of X and the unit’s digit of Y.
E.g: Let the number is 7123
Unit digit = 1
Tens digit = unit digit in 7 × 3 = 21 i.e., 1.
So, last two digit = 11.
Case-2: If unit digit in X is 3, 7 or 9
In this case first we convert X to X1 such that the unit’s digit in X1 is 1, then we use Case 1.
Y  Y 
Y 4  4   
In case of 3 and 7: X will be converted to X1 = X4 so that
= X (=
X ) ( X1 ) 4  .
Y  Y 
Y 2  2   
In case of 9: X will be converted to X1 = X2 so that
= X (=
X ) ( X1 ) 2  .
E.g: Last two digit of 1728

Q Unit digit of 17 is 7
28
=
So 17 28 (17
= 4 4
) (83521)7
Unit’s digit of 1718 = Unit digit of (83521)7 = 1
& Ten’s digit of 1718 = unit digit of (2 × 7) = Unit digit of (14) = 4
Case-3: If unit digit in X is 2, 4, 6 or 8
In this case we use the following identities.
(i) 210 ends in 24.
(ii) 24(odd +ve no.) ends in 24.
(iii) 24(even +ve no.) ends in 76.
(iv) 76n, where n is any natural number, ends in 76.

20 Class-X MATHEMATICS P
W
E.g: 21024 = 21020 × 24 = (210)102 × 24

Now, 210 end in 24 and any number, which ends in 24, raised to power 102 (even) ends in 76. Thus, (210)102 ends in 76.
Also, 24 ends 16.
Thus, Last two digit of 21024 = Last two digit of 76 × 16 = 16
Case-4: If unit digit in X is 5
In this case we use the below table:
Tens digit of the number Unit’s digit of the number Last two digits
Even number Odd number 25
Even number Even number 25
Odd number Odd number 75
Odd number Even number 25

E.g: Find the last two digit of (235)237


Sol. Q Digit at ten’s place is 3 which is odd & the power is also odd
So, (235)237 ends in 75.

Identities Related to an + bn or an – bn
(i) an – bn is always divisible by a – b.
(ii) an – bn is divisible by a + b if n is even.
(iii) an + bn is divisible by a + b is n is odd.
E.g: Find the remainder when 1516 – 616 is divided by 21.
Sol. Q n is even, so 15 + 6 = 21 divides 1516 – 616. Hence remainder is 0.

Highest Power of a Number in a Factorial


U Factorial: Factorial of a non-negative integer n is denoted by n! and defined as, n! = n(n – 1)(n – 2)(n – 3)... 3.2.1
E.g: 5! = 5 × 4 × 3 × 2 × 1; 2! = 2 × 1
Note: 0! = 1
U Greatest integer function: Greatest integer function of a real number x is denoted by [x] and defined as the greatest
integer less than or equal to x.
E.g: [3.2] = 3; [3] = 3; [–1.5] = – 2
U Highest power of a prime in a factorial: Let p is a prime number, then the highest power of p in n!
n  n   n   n 
=   +  2  +  3  +  4  +… , where [ ] is greatest integer function.
 p  p   p   p 
U Highest power of a composite number in a factorial: No. of highest power of a composite number a in n! is
equal to the no. of highest power of the highest prime factor of ‘a’ in n!.
 Highest power of p in n!
NOTE: Highest power of pa, where p is prime, in n! is equal to   , were [ ] is greatest integer
function.  a 

Real Numbers 21
Examples:
 40   40   40 
(i) Maximum power of 3 in 40! =   +  2  +  3  = [13.33] + [4.44] + [1.48] = 13 + 4 + 1 = 18
 3  3  3 
(ii) Maximum power of 6 in 50!
 6= 2 × 3 , here 3 is highest prime factor of 6.

 50   50   50 
⇒ Maximum power of 6 in 50! = Maximum power of 3 in 50 ! =   +  2  +  3  = [16.66] + [5.55] + [1.85]
 3  3  3 
= 16 + 5 + 1 = 22
 Maximum power of 3 in 40! 18 
= 
(iii) Maximum power of 9 in 40!
2
=  [9]
  2= = 9

Excluded from NCERT

255 = 102 × 2 + 51
Exercise-I
102 = 51 × 2 + 0
1. Use Euclid’s division algorithm to find the HCF
\ HCF (867, 255) = 51
of
2. Show that any positive odd integer is of the form
(a) 135 and 225
6q + 1, or 6q + 3, or 6q + 5, where q is some
(b) 196 and 38220
integer.
(c) 867 and 255
Sol. According to Euclid’s Division Lemma if we have
Sol. (a) 135 and 225 two positive integers a and b, then there exist
As we can see, 225 is greater than 135. Hence, unique integers q and r which satisfies the condition
on applying Euclid’s division algorithm, we a = bq + r where 0 ≤ r < b.
have
Let x be any positive odd integer which when
225 = 135 × 1 + 90 divided by 6 gives q as quotient and r as remainder.
135 = 90 × 1 + 45 According to the Euclid’s division lemma
90 = 45 × 2 + 0 x = 6q + r where 0 ≤ r < 6.
We have obtained zero as remainder. The So r can be either 0, 1, 2, 3, 4 and 5.
divisor of this step becomes our H.C.F.
Therefore, x = 6q or 6q + 1 or 6q + 2 or 6q + 3 or
Hence, the HCF of 135 and 225 is 45. 6q + 4 or 6q + 5.
(b) By Euclid’s Division Algorithm, we have
Case-1: 6q + 0: 6 is divisible by 2, so it is an even
38220 = 196 × 195 + 0 number.

r = 0 so H.C.F. will be 196. Case-2: 6q + 1: 6 is divisible by 2, but 1 is not

\ HCF (38220, 196) = 196. divisible by 2 so it is an odd number.
(c) By Euclid’s Division Algorithm, we have Case-3: 6q + 2: 6 is divisible by 2, and 2 is also
867 = 255 × 3+ 102 divisible by 2 so it is an even number.

22 Class-X MATHEMATICS P
W
Case-4: 6q + 3: 6 is divisible by 2, but 3 is not Case-1: a = 3q
divisible by 2 so it is an odd number. ⇒ a2 = 9q2
Case-5: 6q + 4: 6 is divisible by 2, and 4 is also = 3 × 3q2
divisible by 2 so it is an even number. = 3m (Assuming m = q2)
Case-6: 6q + 5: 6 is divisible by 2, but 5 is not Case-2: a = 3q + 1
divisible by 2 so it is an odd number.
⇒ a2 = (3q + 1)2 = 9q2 + 6q + 1
And therefore, any odd integer can be expressed in = 3(3q2 + 2q) + 1
the form 6q + 1 or 6q + 3 or 6q + 5.
= 3m + 1 (Assuming m = 3q2 + 2q)
3. An army contingent of 616 members is to
Case-3: a = 3q + 2
march behind an army band of 32 members in
a parade. The two groups are to march in the ⇒ a2 = (3q + 2)2 = 9q2 + 12q + 4
same number of columns. What is the maximum = 9q2 + 12q + 3 + 1
number of columns in which they can march? = 3(3q2 + 4q + 1) + 1
Sol. Given, = 3m + 1. (Assuming m = 3q2 + 4q + l)
Number of members in army contingent = 616 Therefore, the square of any positive integer
(say, a2) is always of the form 3m or 3m + 1. Hence,
Number of members in army band = 32
proved.
If the two groups have to march in the same number
of columns, we need to find out the highest common 5. Use Euclid’s division lemma to show that the
factor (HCF) between the two groups. cube of any positive integer is of the form
9m, 9m + 1 or 9m + 8.
We shall find the H.C.F using Euclid’s algorithm
Sol. Let us suppose, x be any positive integer and
As, 616 is greater than 32, therefore divisor = 3.
616 = 32 × 19 + 8 On using Euclid’s division algorithm,
32 = 8 × 4 + 0 x = 3q + r, where q ≥ 0 and 0 ≤ r < 3. Therefore
Now, we have obtained zero as remainder. r = 0, 1, 2.
Therefore, HCF (616, 32) = 8. Now, on putting the value of r, we get
Hence, the maximum number of columns in which x = 3q (if r = 0)
the two groups can march is 8. x = 3q + 1 (if r = 1)
4. Use Euclid’s division lemma to show that the x = 3q + 2 (if r = 2)
square of any positive integer is either of the On cubing both sides of all the three above
form 3m or 3m + 1 for some integer m. expressions, we get
[Hint: Let x be any positive integer then it is of Case (i): When x = 3q
the form 3q, 3q + 1 or 3q + 2. Now square each x3 = (3q)3 = 27q3
of these and show that it can be rewritten in the = 9(3q3) = 9m;  [where m = 3q3]
form 3m or 3m + 1.] Case (ii): When x = 3q + 1
Sol. Let a be a positive integer, q be the quotient and r x3 = (3q + 1)3
be the remainder.
= (3q)3 + 13 + 3 × 3q × 1(3q + 1)
Dividing a by 3 using the Euclid’s Division Lemma, = 9(3q3 + 3q2 + q) +1
We have: = 9m + 1; [where m = 3q3 + 3q2 + q]
a = 3q + r, where 0 ≤ r < 3 Case (iii): When x = 3q + 2
Putting r = 0, 1 and 2, we get: x3 = (3q + 2)3
a = 3q (if r = 0) = (3q)3 + 23 + 3 × 3q × 2(3q + 2)
a = 3q + 1 (if r = 1) = 9(3q3 + 6q2 + 4q) + 8
a = 3q + 2 (if r = 2) = 9m + 8 [where m = 3q3 + 6q2 + 4q]

Real Numbers 23
Hence, the cube of any positive integer is of the 29 29
=
(e)
form 9m, 9m + 1 or 9m + 8. 343 73
Here, denominator = 73, which is not of the
Exercise-2 29
form 2n × 5m. So, the rational number has
1. Without actually performing the long division, 343
a non-terminating repeating decimal expansion.
state whether the following rational numbers will
have a terminating decimal expansion or a non- 23
(f )
terminating repeating decimal expansion: 2 × 52 3

17 Here, denominator = 23 × 52, which is of the form


13
(a) (b)
3125 8 2n × 5m (n = 3, m = 2). So, the rational number
(c) 64 (d) 15 23
has a terminating decimal expansion.
455 1600 2 × 52
3

29 23 129
(e) (f) (g) 2 7 5
343 (23 52 ) 2 ×5 ×7
129 6 Here, denominator = 22 × 57 × 75, which is not
(g) 2 7 5 (h)
(2 5 7 ) 15 of the form 2n × 5m. So, the rational number
35 77
129
(i) (  j) has a non-terminating repeating
50 210 2 × 57 × 75
2

decimal expansion.
13 13 6 2
Sol. (a) = (h)
=
3125 55 15 5
Here, denominator = 55, which is of the form Here, denominator = 51, which is of the form
2n × 5m (n = 0, m = 5). So, the rational number 2n × 5m (n = 0, m = 1). So, the rational number
13 6
has a terminating decimal expansion. has a terminating decimal expansion.
3125 15
17 17 35 35 7
(b) = (i)= =
8 23 50 2 × 5 2
2×5
Here, denominator = 23, which is of the form Here, denominator = 2 × 5, which is of the form
2n × 5m (n = 3, m = 0). So, the rational number 2n × 5m (n = 1, m = 1). So, the rational number
17 35
has a terminating decimal expansion. has a terminating decimal expansion.
8 50
64 64 77 11 11
(c) = (j) = =
455 5 × 7 × 13 210 30 2 × 3 × 5
Here, denominator = 5 × 7 × 13, which is not Here, denominator = 2 × 3 × 5, which is not of the
of the form 2n × 5m. So, the rational number 77
form 2n × 5m. So, the rational number has a
64 210
has a non-terminating repeating decimal
455 non-terminating repeating decimal expansion.
expansion. 2. Write down the decimal expansions of those
15 3× 5 3 rational numbers which have terminating
(d) = = decimal expansions.
1600 26 × 52 26 × 5
Here, denominator = 26 × 5, which is of the form (a) 13 (b) 17
3125 8
2n × 5m (n = 6, m = 1). So, the rational number
(c) 64 (d) 15
15
has a terminating decimal expansion. 455 1600
1600

24 Class-X MATHEMATICS P
W
23 3. The following real numbers have decimal
(e) 29 (f) expansions as given below. In each case, decide
343 (23 52 )
whether they are rational or not. If they are
129
(g) 2 7 5
(h) 6 rational and of the form
p
, what can you say
(2 5 7 ) 15 q
about the prime factors of q?
35 77 (a) 43.123456789
(i) (  j)
50 210 (b) 0.120120012000120000...
13 13 13 × 25 416 (c) 43.123456789
Sol. (a)
= = =
3125 55 55 × 25 105 Sol. (a) R ational, prime factors of q will be either 2
13 416 or 5 or both only.

⇒= = 0.00416
3125 100000 (b) Not rational.
(b) 2.125 (d) 0.009375 (f) 0.115 (h) 0.4 (i) 0.7 (c) Rational, prime factors of q will also have a
factor other than 2 or 5.

1. The decimal expansion of the rational number 987


5. Statement-I: will have terminating decimal
14587 10500
, will terminate after: [Exemplar]
1250 expansion.
(a) One decimal place
Statement-II: For any natural number a and b there
exist unique whole numbers q and r such that, a = bq
(b) Two decimal places
+ r, (0 ≤ r ≤ b).
(c) Three decimal places
(a) Only I is true (b) Both I & II is true
(d) Four decimal places (c) Only II is true (d) Neither I or II is true
2. If the HCF of 65 and 117 is in the form 6. Select the one having a terminating decimal
65m – 117, then the value of m is: expansion:
[Exemplar CBSE 2019] 77 22
(a) (b)
(a) 4 (b) 2 210 7
(c) 1 (d) 3 23 125
(c) (d)
3. (n2 – 1) is divisible by 8, if n is: [Exemplar] 8 441
(a) An integer (b) A natural number 7. For some integer q, every odd integer is of the form:
 [Exemplar]
(c) An odd integer (d) An even integer
(a) q (b) q + 1
4. Decimal expansion of a rational number is 327.7081. (c) 2q (d) 2q + 1
a
When the number is expressed in form, where 8. Among the following rational numbers, which one
b has a non-terminating repeating decimal expansion?
a & b are co-primes what would be in the prime
45 71
factors of b? (a) (b)
3125 512
(a) 2 and 7 (b) 3 and 8
24
(c) 2 and 5 (d) 2, 3 and 5 (c) (d) None of these
200

Real Numbers 25
9. If a is a rational number, then 52a – 22a is divisible by: 19. A certain number has exactly eight factors including
(a) Both 3 and 7 (b) 9 1 and itself. Two of its factors are 21 and 35. The
(c) 3 (d) None of these number is:
(a) 105 (b) 210
10. The number 313 – 310 is divisible by:
(c) 420 (d) 525
(a) 3 and 5 (b) 2, 3 and 13
20. The last digit in the finite decimal representation of
(c) 2, 3 and 10 (d) 3 and 10 2004
1
11. Find the smallest number which on being divided the number   is:
5
by 15, leaves 5 as remainder, on being divided by
25, leaves a remainder of 15 and on being divided (a) 2 (b) 4
by 35 leaves a remainder of 25. (c) 6 (d) 8
(a) 1050 (b) 525 21. What is the greatest positive integer ‘n’ which
(c) 540 (d) 515 makes n3 + 100 divisible by ‘n + 10’?
(a) 890 (b) 810
12. The number 10A – 1 is divisible by 11 for:
(c) 1000 (d) 900
(a) All value of A
22. Which of the following can never bе а common
(b) Even values of A
factor of 287 + x and 378 + x where x is any natural
(c) Odd values of A
number?
(d) A must be multiple of 11 (a) 26 (b) 13
13. A six digit number which consists of only one type (c) 91 (d) 7
of digits, either 1, 2, 3, 4, 5, 6, 7, 8 or 9, eg 111111, 23. Number 1146600 can be written as the product of
222222... etc. This six digit number is always two factors, in how many ways?
divisible by: (a) 100 (b) 273
(a) 11 (b) 13 (c) 216 (d) 108
(c) 7 (d) All of these 24. Find unit’s digit in a = 717 + 734.
14. The difference of the squares of two odd natural (a) 7 (b) 8
numbers is divisible by: (c) 10 (d) 6
(a) 8 (b) 6
25. Last two digits of 33288 will be:
(c) 14 (d) 16
(a) 41 (b) 81
15. For any positive integer a, a3 – a is divisible by: (c) 56 (d) 93
(a) 2 (b) 6
26. Find the unit’s digit of (90 + 91 + 92 + 93 + ... + 92009).
(c) 4 (d) None of these
(a) 9 (b) 5
16. Find the number of natural numbers lying between (c) 0 (d) 7
200 and 400 which are divisible by 4 and 5.
(a) 10 (b) 11 27. Find the greatest power of 7 contained in 926!.
(c) 9 (d) None of these (a) 148 (b) 1078
17. Find the number of perfect cubes in the sequence (c) 152 (d) None of these
11, 22, 33, 44, ..., 100100. 28. How many zeroes will be there at the end of 36!36!.
(a) 32 (b) 34 (a) 6! (b) 8 × 36!
(c) 37 (d) 40
(c) 36! (d) 83 × 36!
18. Find the largest number which will divide 398,
436 and 542 and leave 7, 11 and 15 as remainders, 29. Find the value of n for which 2200 – 2192.31 + 2n is
respectively. a perfect square.
(a) 17 (b) 16 (a) 198 (b) 208
(c) 20 (d) 19 (c) 232 (d) 146

26 Class-X MATHEMATICS P
W
Competitive Corner

1. When 31513 and 34369 are divided by a certain (a) 7 (b) 2


three digit number, the remainders are equal, then (c) 8 (d) 11
the remainder is .... [Andhra Pradesh 2017]
10. The expression 14m – 6m will always divisible by
(a) 86 (b) 97
[Delhi 2018]
(c) 374 (d) 113
(a) 8 (b) 20
2. The sum of squares of two consecutive even (c) 14 (d) 6
numbers added by 4 is always divisible by:
[Chandigarh 2017] 11. How many natural numbers between 15 to 500
when divided by 6 leave remainder 5?
(a) 24 (b) 16
[Maharashtra 2018]
(c) 8 (d) 32
(a) 80 (b) 81
3. If n is a natural number the 92n – 42n is always
divisible by: [Delhi 2017] (c) 82 (d) 83
(a) 13 (b) both 5 and 13 12. The largest number which divides 72 and 127
(c) 5 (d) None of the above leaving remainders 7 and 10 respectively is:
4. A number when divided by 5, 3 and 2 leaves [Gujrat 2018]
remainders 4, 2 and 1 respectively. Out of all three (a) 845 (b) 458
digit numbers, find the total such numbers. (c) 65 (d) 13
[Haryana 2017]
(a) 28 (b) 29 13. If 102017– 2017 is expressed as integer, what is the
sum of its digits? [Telangana 2018]
(c) 30 (d) 31
(a) 18144 (b) 17468
5. A positive integer n when divided by 9, gives 7
as remainder. What will be the remainder when (c) 16466 (d) 18564
(3n – 1) is divided by 9? [Jharkhand 2017] 14. If 39 + 312 + 315 + 3n is a perfect cube (of an integer)
(a) 1 (b) 2 where n ∈ N, then find the value of n.
(c) 3 (d) 1 [Telangana 2018]
6. What is the smallest number which leaves the same (a) 18 (b) 14
remainder 1 on division by 18, 24, 30, 42? (c) 16 (d) 17
[Kerala 2017] LCM of (1, 2,3,..., 200)
(a) 2519 (b) 2520 15. Compute:
LCM of (102,103,104,..., 200)
(c) 2521 (d) 2522 [Telangana 2018]
7. What is sum of all factors of 256? [Kerala 2017] (a) 101 (b) 106
(a) 511 (b) 512 (c) 184 (d) 176
(c) 1023 (d) 1024 16. Product of any three consecutive even numbers is
8. Which is unit digit of 618– 510? [Rajasthan 2017] divisible by: [Madhya Pradesh 2018]
(a) 5 (b) 8 (a) 2 (b) 4
(c) 1 (d) 9 (c) 16 (d) 12
9. If a natural number ‘a’ is divided by 7, the remainder 17. What is the ascending order of the surds 3 2, 6 3, 9 4 ?
is 5. If a natural number ‘b’ is divided by 7, the  [Orrisa 2018]
remainder is 3. The remainder is ‘r’ if a + b is divided
(a) 9
4, 6 3, 3 2 (b) 9
4, 3 2, 6 3
3r + 5
by 7. Find the value of . [Chandigarh 2018]
4 (c) 3
2, 6 3, 9 4 (d) 6
3, 9 4, 3 2

Real Numbers 27
18. The unit digit in the expression 55725 + 735810 + 27. When a natural number x is divided by 5, the
22853 is: [Bihar 2019] remainder is 2. When a natural number y is divided
(a) 0 (b) 4 by 5, the remainder is 4. The remainder is z when
(c) 5 (d) 6 2z − 5
x + y is divided by 5. The value of is:
19. What must be substracted from 21, 38, 55, 106 so 3
that the remainders are proportional?[Orissa 2019]  [NTSE 2015]
(a) 8 (b) 6 (a) –1 (b) 1
(c) 4 (d) 2 (c) –2 (d) 2
20. The product of two 2-digit numbers is 2160 and
28. On dividing a natural number by 13, the remainder
their H.C.F is 12. Then sum of the number is:
is 3 and on dividing the same number by 21, the
[Delhi 2019]
remainder is 11. If the number lies between 500 and
(a) 72 (b) 84
600, then the remainder on dividing the number by
(c) 96 (d) 60
19 is: [NTSE 2016]
21. A number when divided by 6, gives remainder 3, (a) 4 (b) 6
then what will be the remainder if square of that
number is divided by 6. [Punjab 2020] (c) 9 (d) 13
(a) 1 (b) 2 29. The sum of all the possible remainders which can
(c) 3 (d) 4 be obtained when the cube of a natural number is
22. If ‘a’ and ‘b’ are any two positive integers and divided by 9, is: [NTSE 2017]
ab × ba = 72, then the values of a and b are: (a) 5 (b) 6
[Tamil Nadu 2020] (c) 8 (d) 9
(a) 2, 3 (b) 3, 2 30. On dividing 2272 as well as 875 by a 3-digit
(c) 2, 3 or 3, 2 (d) 3, 3 number N, we get the same remainder in each case.
23. When (1012 – 1) is divided by 111 the quotient is: The sum of the digits of N is [NTSE 2017]
 [Uttar Pradesh 2020] (a) 10 (b) 11
(a) 9009009 (b) 9009009009 (c) 12 (d) 13
(c) 9000009 (d) 900000009
1
24. If [n] denotes the greatest integer < n and (n) denotes 31. Given that = 0.142857 which is a repeating
7
the smallest integer > n; n being a real number, then
decimal having six different digits. If x is the sum
 1   1  1   1
1  × 1  − 1  ÷ 1  + (1.5) is: of such first three positive integers n such that
 5   5  5   5
1
[West Bangal 2020] = 0. abcdef , where a, b, c, d, e and f are different
n
(a) 1.5 (b) 2
digits, then the value of x is: [NTSE 2018]
(c) 2.5 (d) 3.5
(a) 20 (b) 21
25. For positive x and y, the LCM is 225 and HCF is
(c) 41 (d) 42
15. There __________. [NTSE 2014]
(a) Is exactly one such pair 32. Which of the following digits is ruled out in the units
(b) Are exactly two such pairs place of 12n + 1 for every positive integer n?
(c) Are exactly three such pairs [NTSE 2018]
(d) Are exactly four such pairs (a) 1 (b) 3
26. LCM of two numbers x and y is 720 and the LCM (c) 5 (d) 7
of numbers 12x and 5y is also 720. The number y 33. If m = n2 – n, where n is an integer, then m2 – 2m
is: [NTSE 2015] is divisible by: [NTSE 2019]
(a) 180 (b) 144 (a) 20 (b) 24
(c) 120 (d) 90 (c) 30 (d) 16

28 Class-X MATHEMATICS P
W
34. On dividing a natural number x by 11, the remainder 35. If 10025–25 is written in decimal notations, then the
is 3, and on dividing x by 17, the remainder is 9. sum of its digits is: [IOQJS 2020]
If the number x lies between 300 and 400, then the (a) 444 (b) 442
remainder on dividing x by 21 is: [NTSE 2020] (c) 424 (d) 422
(a) 9 but not 11 (b) 11 but not 9
(c) both 9 and 11 (d) neither 9 nor 11

ANSWER KEY

Multiple Choice Questions


1. (d) 2. (b) 3. (c) 4. (c) 5. (a) 6. (c) 7. (d) 8. (d) 9. (a) 10. (b)
11. (d) 12. (b) 13. (d) 14. (a) 15. (b) 16. (c) 17. (b) 18. (a) 19. (a) 20. (c)
21. (a) 22. (a) 23. (d) 24. (d) 25. (a) 26. (c) 27. (c) 28. (b) 29. (a)

Competitive Corner
1. (b) 2. (c) 3. (b) 4. (b) 5. (b) 6. (c) 7. (a) 8. (c) 9. (b) 10. (a)
11. (b) 12. (d) 13. (a) 14. (b) 15. (a) 16. (c) 17. (a) 18. (d) 19. (c) 20. (c)
21. (c) 22. (a) 23. (b) 24. (b) 25. (b) 26. (b) 27. (a) 28. (a) 29. (d) 30. (a)
31. (c) 32. (a) 33. (b) 34. (a) 35. (a)

Real Numbers 29
Explanations

5. (c) Number when divides 70 and 125 leaves


School Level remainders 5 and 8, then
70 – 5 = 65
125 – 8 = 117
Quick Recall then HCF of 65 and 117 is
65 = 5 × 13
Assertion & Reason Type Questions
117 = 3 × 3 × 13
1. (d) As, H.C.F × L.C.M = Product of two numbers Hence, HCF of 65 and 117 is 13.
Product of two numbers 3072 13 is the largest number which divides 70 and 125
\ L.C.M = =
H.C.F 16 and leaves remainders 5 and 8.
= 192 ≠ 162 6. (c) Statement-I: As a = x3y2, b = xy3
Here reason is true [standard result] HCF (a, b) = xy2
2. (c) Here, reason is false as 16 = ±4, which is not Hence, statement I is incorrect.
an irrational number. Statement-II: 12n = (22 × 3)n
3. (c) We have, LCM (p, q) × HCF (p, q) = p × q Prime factorisation of 12n does not contain prime
no. 5.
Assertion is correct
7. (d) HCF × LCM = product of numbers
LCM × 5 = 150 27 × 162 = 54 × a ⇒ a = 81
150 8. (d) As the LCM is the product of the highest exponent
⇒ LCM = = 30 ≠ 40
5 of each prime number in prime factorisation of given
\ Reason is false. numbers (a, b, c), as in LCM(a, b, c) the exponent
of 3 is 2 which is possible only if p = 2.
9. (b) As, 95 = 5 × 19 and 171 = 3 × 3 × 19
Multiple Choice Questions
HCF (95, 171) = 19 m
1. (c) HCF of two numbers is a factor of LCM of the 10. (c) LCM (16,20,24) = 240 = 2 × 2 × 2 × 2 × 3 × 5
numbers Perfect Square = LCM × 15 = 3600
2. (a) The power of 2 and 5 in prime factorisation are 11. (a) HCF × LCM = product of numbers
3 and 2 respectively. 2 × 36 = 18 × a
So, number of zero in the number = power of 10 a=4
= minimum (power of 2 and power of 5) = 2
3. (d) The least number divisible by all the numbers Subjective Questions
from 1 to 10 will be the L.C.M of the following
number: 1, 2, 3, 4, 5, 6, 7, 8, 9, & 10. Very Short Answer Type questions
So, the L.C.M. of these number is 1. Q HCF × LCM = Product of numbers
1 × 2 × 2 × 2 × 3 × 3 × 5 × 7 = 2520 Product of numbers
⇒ LCM =
Hence, the least number divisible by all the numbers HCF
from 1 to 10 is 2520 336 × 54
⇒ LCM = = 56 × 54
4. (c) p = a × b × b; q = a × a × a × b 6
Therefore, L.C.M of p and q = a3b2. ⇒ LCM = 3024

30 Class-X MATHEMATICS P
W
2. 2 1080 Required number = Multiple of 840 which is nearest
and greater than 100000.
2 540 1080 divided by 2 gives 540
= 100800.  [ 100800 = 840 × 120].
2 270 540 divided by 2 gives 270 2. To find the maximum number of students in a
3 135 270 divided by 2 gives 135 group, we need to find H.C.F of 437 and 342.
H.C.F (437, 342) = 19
3 45 135 divided by 3 gives 45
437 342
\ number of groups = + = 41 groups
3 15 45 divided by 3 gives 15 19 19
5 5 15 divided by 3 gives 5 3. Since 3823a is obtained after multiplying 484 with
⸫ 1080 =2×2×2×3×3×3×5 a certain number.
= 23 × 33 × 5 It is clear that 3823a is divisible by 484. Therefore
3. r = 17 × 2 = 34 484 is a factor of 3823a.
q = r × 2 ⇒ 34 × 2 = 68 The prime factorisation of number 484:
and p = q × 2 ⇒ 68 × 2 = 136 484 = 2 × 2 × 11 × 11 = 22 × 112
4. Q Only 1 and the prime number itself divides the Therefore, 3823a is divisible by both 22 and 112.
prime number. Also, we know that a no. is divisible by 4 if the no.

So, there are only two factors 1 and the prime formed by its last two digit is divisible by 4.
number itself. So, 3823a to be divisible by 4, a must be 2 or 6.
5. The prime factorisation of given number 6n: So, the possible numbers are 38232 and 38236.
6n = (2 × 3)n = 2n × 3n Out of which only 38236 is divisible by 112 as
For the number 6n to be divisible by 7, the prime 38236 = 22 × 112 × 79
factorisation of 6n should contain the prime number 7. Therefore, value of a is 6.
As prime factorisation of 6n contains only numbers 2 4. First 100 multiples of 10 are 10, 20, 30, 40, ..., 990,
and 3.
1000.
\ 6n is not divisible by 7.
We have to find the number of zeroes in
6. Required number = LCM (4, 5 and 6) + 3 10 × 20 × 30 × ..... 990 × 1000.
= 60 + 3 = 63 It can be written as:
7. The prime factorisation of 429 can be written as- 10100(1 × 2 × 3 × 4 × ... × 99 × 100)
429 = 3 × 11 × 13. Now the number of zeroes in (1 × 2 × 3 × 4 × ... ×
8. Given L.C.M. (150, 100) = 300 99 × 100).
Q H.C.F × L.C.M = Product of two numbers As we know that the trailing zero is formed when
Product of 150 and 100 a multiple of 5 is multiplied with a multiple of 2.
\H.C.F. (150, 100)= Now, all we have to do is count the number of 5’s
L.C.M.(150,100)
and 2’s in the multiplication.
150 × 100
= = 50 Let’s count the 5’s first 5, 10, 15, 20, 25 and so on
300
making a total of 20. However, there is more to this.
9. A rational number between 2 = 1.41 and
Since 25, 50, 75 and 100 have two 5’s in each of
3 = 1.73 are 1.42, 1.52 etc. them (25 = 5 × 5, 50 = 2 × 5 × 5, …), you have to
count them twice. This makes the grand total 24.
Short Answer Type Questions Moving on to count the number of 2’s. 2, 4, 6, 8, 10
1. 8 = (23), and so on. Total of 50 multiples of 2’s, 25 multiples
15 = (3)(5), of 4’s (count these once more), 12 multiples of 8’s
(count these once more) and so on… The grand
21 = (3)(7)
total comes out to 97.
\ LCM of (8, 15, 21) = (23)(3)(5)(7) = 840.

Real Numbers 31
Each pair of 2 and 5 will cause a trailing zero. Since 5a − 2b
we have only 24, 5’s, we can only make 24 pairs ⇒ 3=
b
of 2’s and 5’s thus the number of trailing zeros in
Q a, b are integers
(1 × 2 × 3 × 4 × ... × 99 × 100) is 24.
Therefore, the total number of zeros in the product 5a − 2b
∴ is a rational number
= 100 + 24 = 124 b
\ 124 number of zeroes will be there at the end of ⇒ √3 is a rational number
the product. which contradicts the fact that 3 is irrational
5. As we know that,
2+ 3
2 hours = 120 minutes Thus, is an irrational number.
5
4 hours = 240 minutes
5.5 hours = 330 minutes 9. Let us suppose, to the contrary that 3 − 5 is a
We need to find the time after which all the three rational number i.e.,
people will meet again at the starting point. For this, a
3− 5 = , (a and b are co-prime and b ≠ 0)
there will be a number that is divisible by 120, 240 b
and 330 and that will be the time when all the three a
⇒ 5= 3 −
meet again at the starting point. To find this we b
a
have to take LCM of all three numbers. Since, a and b are integers, So, 3 − is a rational
b
L.C.M. (120, 240 and 330) = 2640 minutes number.
2640 ⇒ 5 is also a rational number.
Time = hours = 44 hours.
60
6. We need to find the L.C.M of 7, 8, 11 and 12. But, this contradicts the fact that 5 is irrational.
L.C.M (7, 8, 11, 12) = 1848 Hence, 3 − 5 is an irrational number.
Therefore, after every 1848 seconds bells will ring 10. Let’s assume, to the contrary, that 3 is rational.
together. a
Then 3= where a and b are co-primes.
In 3 hours, there are 3600 × 3 = 10800 secs. b
Number of times the bell would ring So, 3b = a.
10800 On squaring both sides, we get
= = 5.84
1848 a2 = 3b2 \ 3 divides a2.
Therefore, bells will ring together 5 times in the Now, using Theorem 2, it follows that 3 divides a.
next 3 hours. So, we can write a = 3c for some integer c.
7. Since, 404 = 22 × 101 and 96 = 25 × 3 Again on squaring both sides, we get
HCF (404, 96) = 22 = 4 a2 = 9c2
⇒ 3b2 = 9c2[ a2 = 3b2]
LCM (404, 96) = 101 × 25 × 3 = 9696
⇒ b2 = 3c2
Now, HCF × LCM = 4 × 9696 = 38784 This means that 3 divides b2 and therefore 3 divides
Also, 404 × 96 = 38784 b (again using Theorem 2).
Thus, HCF × LCM = Product of 404 and 96. Therefore, a and b have at least 3 as a common
factor.
8. Let us suppose, to the contrary, 2 + 3 be a rational But this contradicts the fact that a and b do not have
number. 5
a common factor other than 1.
2+ 3 a This contradiction has arisen because of our wrong
⇒ = (b ≠ 0, a and b are co-prime and
5 b assumption that 3 is rational.
integers) So, we conclude that 3 is irrational.

32 Class-X MATHEMATICS P
W
11. Consider that p + q is rational. 2. Let us assume that 7 + 11 is rational number.

p + q = a, where a is rational. a
Now Let us 7 + 11 = (a and b are co-prime, integer
b
q =a– p and b ≠ 0)
Therefore,
a
By squaring on both sides, we get \ 7 = − 11
b
q = a2 + p – 2a p
Squaring on both sides, we get
Using the algebraic identity (a – b)2 = a2 + b2 –
a2 a
2ab 7= + 11 − 2 11
2
b b
a2 + p − q Rearranging
p = , which is a contradiction as the
2a a2
a
right hand side is a rational number while p is 2 11 = 2 + 11 − 7
b b
irrational, since p is a prime number.
a + 4b 2 b a 2 + 4b 2
2

Therefore, p + q is irrational. 11 = × =
b2 2a 2ab

Long Answer Type Questions a 2 + 4b 2


Since, a, b are integers is rational number.
2ab
1. If a = c then
So 11 is rational number.
a + b =c + d ⇒ b = d ⇒ b =d
This contradicts the fact that 11 is irrational
If a ≠ c then there exist a rational number k such number.
that a = c + k
Hence 7 + 11 is irrational number.
Now, a + b =+
c d
⇒c+k + b =c+ d Cased Study Type Questions
⇒ k + b =d ..(i) Case Study-I
On squaring both sides, we get: 1. (d) Total number of participants

( )=
( d)
2 2 = 60 + 84 + 108 = 252
⇒ k+ b
2. (a) L.C.M of 60, 84 and 108 is:
⇒ k 2 + 2 bk + b =d
60 = 22 × 3 × 5
2
⇒ d −k −b =2k b
84 = 22 × 3 × 7
2
d −k −b
⇒ b= 108 = 22 × 33
2k
LCM(60, 84, 108) = 22 × 33 × 5 × 7 = 3780
 d, k, b are rational numbers
3. (d) If in a room all of the participants are of same
d − k 2 − b2 subject then maximum number of participants to be
\ is rational
2k seated is HCF of 60, 84 and 108.
⇒ b is rational. First we find the HCF of 60 and 84 by using
\ b is the square of a rational number. Euclid’s Division algorithm.
From equation (i), we have 84 = 60 × 1 + 24

d= k + b 60 = 24 × 2 + 12
24 = 12 × 2 + 0
⇒ d is rational
So, HCF (84, 60) = 12
\ d is the square of rational number
Now, we will find the HCF of 12 and 108.
Therefore, either a = c and b = d; or b and d are the
squares of rationals. 108 = 12 × 9 + 0

Real Numbers 33
⸫ HCF (12, 108) = 12
Hence, HCF (60, 84 and 108) = 12 Competitive Level
Least number of rooms required for all the
252
participants = = 21 Multiple Choice Questions
12
Case Study-II 14587 14587 × 23
1. (d) ⇒ , terminate after four
1. (a) In one turn, Akbar goes up & comes down 7 & 2 × 54 (10) 4
2 stairs respectively. So, he goes up 5 stairs up in
decimal places.
each turn.
Now, Q Akbar cannot cross 100th stair so the 2. (b) On using Euclid’s division algorithm,
highest stair reached by Akbar will be 95th stairs. In
a = bq + r, 0 ≤ r < b
one turn, Birbal climbs up & gets down 5 & 2 stairs 117 = 65 × 1 + 52
respectively. So, he goes up 3 stairs in each turn.
65 = 52 × 1 + 13
Now, Q Birbal cannot cross 100th stair so the 52 = 13 × 4 + 0
highest stair climbed by Birbal will be 96th stair.
\ H.C.F (65, 117) = 13
In one turn, Mohit climbs up & gets down 10 & 3
stairs respectively. So, he does up 7 stairs in each H.C.F = 65m – 117  [Given]
turn. \ 65m – 117 = 13 ⇒ 65m = 130 ⇒ n = 2
Now, Q Mohit cannot cross 100th
stairs so the 3. (c) Let p = n2 – 1. Now, n can be even or odd
highest stair climbed by Mohit will be 91th stair. Case-1: If n is even i.e., n = 2a, a ∈ Z
Thus, Birbal reaches the nearest point 96th stair. Then, p = (2a)2 – 1 ⇒ p = 4a2 – 1
2. (d) They can meet only at the stairs which are For a = 1
multiple of LCM of 5, 3 & 7 4(1)2 – 1 = 3, not divisible by 8

Q LCM (5, 3, 7) = 105 Case-2: If n is odd i.e. n = 2a + 1, a ∈ Z
So, they can meet at 105 stair but 105th is after 100th Then, p = (2a + 1)2 – 1 = 4a2 + 4a + 1 – 1
stair So, they cannot meet in between on the same = 4a(a + 1)
stair. If a is even then a = 2n, n ∈ z.
⇒ p = 4 × (2n) (2n + 1) = 8n (n + 1) = 8m, where,
95
3. (c) Akbar takes = 19 steps to reach nearest to m = n(n + 1)
5
So, p is divisible by 8
96
hundred, Birbal takes = 32 steps to reach nearest If a is odd then a + 1 is even so, a + 1 = 2n, n ∈ Z
3
91 ⇒ p = 4 × (2n – 1) (2n) = 8n(n – 1) = 8m, where
to hundred & Mohit takes = 13 steps to reach m = n(n – 1)
nearest to hundred. 7
So, p is divisible by 8.
So, Mohit takes least no. of steps to reach near 3277081
hundred. 4. (c)
2 4 × 54
987 47
4. (a) Any two of them can meet of the stairs which are 5. (a) Statement-I: 3 2
= 2 3
multiple of LCM (3, 5), LCM (5, 7), LCM (7, 3). 3× 7 × 5 × 2 2 ×5
987

Q LCM (3, 5) = 15 is the least of 15, 35 & 21. So, hence, is terminating decimal.
10500
15th stair is 1st stair at which Birbal & Akbar will
Statement-I is true.
meet.
Statement-II: As according to the Euclid’s Division
5. (c) Q 21 is second least of 15, 21 & 35. Lemma,
So, 21th is 2nd stair at which two of them (Birbal a = bq + r, where 0 ≤ r < b.
and Mohit) will meet. Hence, statement-II is false.

34 Class-X MATHEMATICS P
W
6. (c) As the denominator 8 is in 23 form, hence its 17. (b) Here, all the numbers having power which are
decimal expansion will be terminated. multiple of 3 will be perfect cube, moreover 1, will
also be a perfect cube.
7. (d) Q a = bq + r, where 0 ≤ r < b.
\ total number of perfect cubes = 33 + 1 = 34.
For b = 2, a = 2q or a = 2q + 1
18. (a) HCF of |398 – 7|, |436 – 11| |542 – 15| is 17.
Q a = 2q + 1 is not divisible by 2,
19. (a) No. of factors of 105 = 8.
So it is an odd integer
21 & 35 are factor of 105.
8. (d) As 3125 = 54
2004 2004
512 = 29 1  2
20. (c)   =  unit digit for 24K, K ∈ N is 6 .
5
  10
 
200 = 2352
21. (a) n + 100 = (n + 10) (n2 – 10n + 100) – 900
3
9. (a) 52a −22a is of the form a2a − b2a which is
If (n + 10) divides n3 + 100 ⇒ (n + 10) divides 900.
divisible by both (a + b) and (a – b).

\ Largest positive divisor n = 900 – 10 = 890
So, 52a − 22b is divisible by both 7, 3.
22. (a) [HINT: 26 is even number & 26 can not divide
10. (b) 313 – 310 = 310(26) = 2 × 13 × 310 odd number.]
11. (d) Among the given options 515 is short by 10 for 23. (d) 1146600 ⇒ 23 × 32 × 52 × 72 × 13
complete division by 15, 25 or 35. Number of factors = (3 + 1) (2 + 1) (2 + 1) (2 + 1)
12. (b) Let A is even then, A = 2a, a ∈ Z (1 + 1) = 216
(10A – 1) = (102a – 1) = 100a – 1a Number of ways in which 1146600 can be written
as product of 2 factors
= (100 – 1)(z), z is some integer.
216
= 99(z), z is some integer. = = 108
2
Therefore, for any even power it is divisible by 11. 24. (d) a = 717 + (7)34
For A = 1 (odd) = (74)4 × 71 + (74)8 × 72.
10A – 1 = 101 – 1 = 9, which is not divisible by 11. Digits at unit’s place = 1 × 7 + 1 × 9 = 7 + 9 = 6
13. (d) Q 111111 = 3 × 7 × 11 × 13 × 37 25. (a) 33288 = (334)72. = (332 × 332)72
So, 111111 is divisible by 7, 11 & 13. = (1089 × 1089)72 = (1185921)72
Also, all the no. of the type 222222, 333333, ....., The unit’s digit of this number will always be 1 as
999999, can be written as multiple of 111111. 1n is 1.
14. (a) Let two odd natural no’s be (2a + 1) and Now to get tens digit, multiply the tens digit of the
(2b + 1), then number (2 here) with the last digit of the exponent
(2 here) i.e., 2 × 2 = 4.
(2a + 1)2 – (2b + 1)2 = 4(a – b)(a + b + 1)
Hence, last two digits of (1185921)72 is 41.
= (4 × (a – b) × (a + b + 1)) is a multiple of 8.
26. (c) Q unit’s digit of 9odd is 9 and of 9even is 1.
15. (b) a3 – a = (a – 1) a(a + 1) is the product of 3
of 90 + 91 + 92 + ...... + 92009
consecutive integers and is divisible by 6.
Unit’s digit = 1 + 9 + 1 + ...... + 9
16. (c) Number which is divisible by 4 and 5 is the
= Unit digit of (1005 × 1 + 1005 × 9)
number divisible by 20.
= Unit digit of (1005 × 10) = 0
So total numbers between 200 and 400 which are
27. (c) The maximum power of 7 in 926!
divisible by 20 is 9 (excluding 200 and 400) i.e. 220
.... 380 = (9) terms.  926   926   926 
= + 2+ 3
\ there are 9 natural numbers between 200 and  7   (7)   (7) 
400, which are divisible by 4 and 5. = 132 + 18 + 2 = 152

Real Numbers 35
28. (b) In 36! power of 5 is given as: So, the possible values of the divisor are d = 119,
238, 357, 476, 714 and by dividing the given
 36   36 
8
 5  +  (5) 2  = numbers by these possible values of the divisors we
    get a remainder of only 97.
And power of 2 is 34. For example:
Therefore, number of zeroes in 36!36! is 8 × 36! 31513 97 34369 97
= 132 = and 144
29. (a) q = 2200 – 2192.31 + 2n 238 238 238 238
= 2192(28 – 31) + 2n 2. (c) 2p, 2p + 2
= 2192 × 225 + 2n (2p2 + 2p + 2)2 + 4
= 2192(225 + 2n – 192) = 8p2 + 8p + 8
= 2192(225 + y) 3. (b) Given, 92n – 42n is in the form of a2n – b2n where
Now, in order to make (225 + y) a perfect square n is a natural number.
the minimum values of y should be: Since a2n – b2n is always divisible by (a – b) and
2n – 192 = 26 ⇒ n – 192 = 6 (a + b), 92n – 42n is always divisible by:
So, n = 192 + 6 = 198 9–4=5
9 + 4 = 13
Competitive Corner \ 92n – 42n is always divisible by both 5 and 13.
4. (b) \ The numbers are : 119, 149, .... 989
1. (b) Given numbers are 31513 and 34369.
989 = 119 + 30 (n + 1)
Let the divisor of the numbers be d and the remainder
be r. 870
(n – 1) =
30
Now we will write both the numbers in the expanded
n – 1 = 29
form of the divisor, quotient and the remainder.
Therefore, we get n = 30
5. (b) Let n = 9q + 7 ⇒ 3n – 1 = 27q + 20
31513 = dn + r …(i)
3n – 1 = 27q + 18 + 2
34369 = dm + r …(ii)
9(3q + 2) + 2 = 3n – 1
Here n and m are the quotients of the numbers.
\ 3n – 1 = 9k + 2
Now we will subtract the equation (i) from the
\ Remainder is 2 when 3n – 1 is divided by 9.
equation (ii). Therefore, we get
6. (c) Taking L.C.M. of 18, 24, 30 and 42 we get;
34369 – 31513 = dm + r – (dn + r)
\ L.C.M = 2 × 2 × 2 × 3 × 3 × 5 × 7 = 2520.
Subtracting the like terms, we get
Thus, The least required number is
⇒ 2856 = dm + r – dn – r 2520 + 1 = 2521.
Now by simplifying it, we get 7. 
(a) The factors of 256 are 1, 2, 4, 8, 16, 32, 64, 128,
⇒ d(m – n) = 2856 and 256.
Now we will make all the possible 3 digit multiples The sum of these factors = 1+ 2 + 4 + 8 + 16 + 32
+ 64+128 + 256 = 511.
for the above equation, we get
The sum of the factors is 511.
d(m – n) = 2856 = 119 × 24
(c) 618 – 510 will end with 6 – 5 = 1 as 6n and 5n end
8. 
d(m – n) = 2856 = 238 × 12 with 6 and 5 respectively.
d(m – n) = 2856 = 357 × 8 9. (b) ‘a’ is divided by 7, the remainder is 5
d(m – n) = 2856 = 476 × 6 ⇒ a = 7m + 5
d(m – n) = 2856 = 714 × 4 ‘b’ is divided by 7, the remainder is 3

36 Class-X MATHEMATICS P
W
⇒ b = 7n + 3 15. 
(a) LCM of (1, 2, 3, ..., 200) = LCM of (101,
a + b = 7m + 5 + 7n + 3 102, ..., 200) (Q 101, 102, ..., 200 contains multiples
of all numbers between 1 to 100)
⇒ a + b = 7m + 7n + 8
⇒ a + b = 7m + 7n + 7 + 1 Also, LCM of (101, 102, ..., 200) = LCM of (102,
103, ..., 200) × 101 (Q 101 is a prime number)
⇒ a + b = 7(m + n + 1) + 1
LCM of (1, 2,3,..., 200)
⇒ a + b = 7k + 1
\
LCM of (102,103, 200)
The remainder is ‘r’ if a + b is divided by 7
101 × LCM of (102,103,..., 200)
⇒ a + b = 7k + r = = 101
LCM of (102,103,..., 200)
⇒r=1
16. (c) Let the no. be: 2n, 2n + 2, 2n + 4
(3r + 5) (3 × 1 + 5)
= (2n) (2n + 2) (2n + 4) = 8(n) (n + 1) (n + 4)
4 4 have must be divisible by 16
(3 + 5) 8 1 1 1
= = =2 17. (a) 3 6 9
4 4 2, 3, 4 = 2 3 , 36 , 4 9

10. (a) 14m – 6m LCM of the denominators of the exponents 3, 6, 9


= 18
an – bn is always divisible by a – b
Multiplying the exponents with 18
14m – 6m is divisible by 8.
(26)1/18, (33)1/18, (42)1/18
11. (b) Number are in the Given → 6K + 5
(64)1/18, (27)1/18, (16)1/18
So list of numbers 17, 23, 29, ...., 497
(64)1/18 < (27)1/18 < (16)1/18
497 = 17 + (N – 1)6
9
⇒ N = 81 4<63<32
12. (d) Number = HCF (72 – 7, 127 – 10) = 13. Ascending order = 9 4, 6 3, 3 2
13. (a) 102017 – 2017 18. 
(d) In terms of finding the unit’s digit of the
expression, (55725 + 735810 + 22853) is same as
If x > 1,
(51 + 732 + 221). The respective units digit for the
x
In 10 – 2017 last 4 digits are always 7, 9, 8 and 3. three parts of the expression would be: 5 + 9 + 2
104 – 2017 = 7983 = 16 → required answer is 6. Option (d) is correct.
105 – 2017 = 97983 19. (c) HCF of (106 – 55), (55 – 38) and (38 – 21) = 17
106 – 2017 = 997983 So, Required no. = 21 – 17 = 4
107 – 2017 = 9997983 20. (c) Let numbers are 12x and 12y.
Thus, the sum of the digits in 10x – 2017 = (x – 4)⋅ 2160
9+7+9+8+3 xy
\ 12x × 12y = 2160 ⇒ =
= 15
12 × 12
Hence, for 102017 – 2017
\ Possible factor of 15 = 3 × 5
Sum of the digits = (2017 – 4)⋅9 + 7 + 9 + 8 + 3
\ Numbers are 12 × 3 = 36 and 12 × 5 = 60
= 2013 × 9 + 7 + 9 + 8 + 3 \ Sum = 36 + 60 = 96
= 18117 + 7 + 9 + 8 + 3 = 18,144 21. (c) Let x = 6q + 3
14. (b) 39 + 312 + 315 + 3n = 39[1 + 33 + 36 + 3n–9] Then, x2 = (6q + 3)2
= 39[1 + 33 + 36 + 3m], where m = n – 9 = 36q2 + 36q + 9
39 is cube of 33, we have to find 3m = 6(6q2 + 6q + 1) + 3
1 + 33 + 36 + 3m = 757 + 3m Thus, when x2 is divided by 6, then remainder = 3.
Put m = 1, 2, 3, 4, ..... 22. (a) The number 72 can be factorized as

\ n = 9 + 5 = 14 72 = 2 × 36

Real Numbers 37
36 = 2 × 18 28. (a) 13 – 3 = 21 – 11 = k
18 = 2 × 9 No. = LCM (13, 21) – k = 536
9=3×3 536 = 19 × 28 + 4 ⇒ r = 4
i.e., 72 = 2 × 2 × 2 × 3 × 3 29. (d) Remainder when cube of any no. is divided by
72 = 23 × 32 9 are 0, 1 or 8.
So, a = 2, b = 3 30. (a) 2272 – 875 = 1397 is divisible by the 3-digit no.
12 3 3 6 and 1397 = 127 × 11.
23. (b) 10 − 1 = (10 − 1)(10 + 1)(10 + 1)
111 111 Sum of the digits on N = 1 + 2 + 7 = 10
999 × 1001 × 1000001 1 1 1
=
= 9 × 1001 × 1000001 31. (c)
= 0.142857;
= 0.076923; = 0.047619
111 7 13 21
= 9009009009 So, x = 7 + 13 + 21 = 41
24. (b) [n] < n (integer); (n) > n (integer) 32. (a) By cyclicity (12)n always gives unit digit 2, 4,
\ Expression 6, 8.
 1   1  1   1 So, 12n + 1 will give out digit 3, 5, 7, 9
1  × 1  − 1  ÷ 1  + (1.5)
 5   5  5   5 So, 1 cannot be unit digit of 12n + 1.
=2×1–2÷1+2=2 33. (b) m2 – 2m = (n – 2)(n – 1)(n)(n + 1)
25. (b) Let numbers are 15x and 15y ⇒ divisible by 24.
Then 15xy = 225 ⇒ xy = 15 either x = 15, y = 1 or 34. (a) Required no. = LCM (11, 17) × 2 – 8 = 366 and
x = 3, y = 5. 366 = 21 × 17 + 9
Hence (x, y) = (1,15) or (15, 1) and (3, 5) or (5, 3) 35. (a) 10025 – 25 ⇒ (102)25 – 25 ⇒ 1050 – 25
26. (b) 5y should divide 720 ⇒ y = 144
⇒ 1000......00
 − 25 = 99999......9975

27. (a) x + y = 5 ((m + n) + 1) +1 ⇒ z = 1 50 zeros 48 times 9
2 ×1 – 5 –3 ⇒ \ sum of digits
⇒ ⇒ −1
=
3 3 ⇒ (48 × 9) + 7 + 5 ⇒ 444

38 Class-X MATHEMATICS P
W
2
CHAPTER
Polynomials

School Level

Polynomials
The word Polynomial is derived from two greek words, namely Poly (meaning “many”) and Nomial (meaning “terms”),
therefore meaning of polynomial = many terms.
An algebraic expression p(x) of the form p(x) = a0 + a1x + a2x2 + ......+ anxn, where a0, a1, a2.......an are real numbers and
all the powers of x are non-negative integers is called a polynomial.
For Example
5 2 3 5
(i) x2 + 9x    (ii) 4 x − 5 x + 8    (iii) 3 x + x + 5
8
Following expressions are not polynomials:
2
(i) 3 x − − 5 , as power of x is –1 i.e., negative.
x
1
(ii) 4 x3 − 3 x + 6 , as power of x is i.e., fraction.
2

Degree of a Polynomial
The highest power (exponent) of variable in any of the terms having non-zero coefficient of a polynomial is called degree
of the polynomial.
For example:
In polynomial 4x3 – 7x6 + 4:
U The power of variable in the term 4x3 is 3.
U The power of variable in the term –7x6 is 6.
U The power of variable in the term 4 is 0.
Since, the highest power of variable is 6, therefore degree of the polynomial 4x3 – 7x6 + 4 is 6.
Let's see some more examples.
(i) The degree of polynomial 5y2 – 6 is 2.
(ii) The degree of polynomial 7x – 13x5 + 4x3 is 5.
(iii) The degree of polynomial 13x + 7 is 1 as x = x1.
Knowledge Hub
The degree of a polynomial with more than one variable can be computed by adding the exponents of each variable
in terms. For example: 7a4 – 3ab4
™ The power of variable in the term 7a4 is 4.
™ The power of variable in the term – 3ab4 is 5. (a has exponent 1, b has 4, so 1 + 4 = 5)

Types of Polynomials
In general, the polynomials are divided into three categories.

Types of Polynomials

Based on number of Based on degree Based on number of


distinct variables terms
Based on degree
Degree Name Example
Not defined Zero 0
0 (non-zero)constant (any real number)
1 Linear x+1
2 Quadratic 2­
x + x + 13
3 Cubic 3
x + 2x – x + 4
Generally, a polynomial of degree n, for n greater than 3, is called a polynomial of degree n, although quartic and quintic
polynomial are sometimes used for degree 4 and 5 respectively.
Based on number of terms
Number of non-zero terms Name Example
1 monomial x2
2 binomial x2 + 1
3 trinomial 2
x +x+1
Based on number of distinct variables
Number of distinct variables Name Example
1 Univariate x+3
2 Bivariate x+y+5
3 Trivariate x+y+z+7
Usually, a polynomial in more than one variable is called a multivariate polynomial. In this chapter, mainly we will
discuss only about the polynomials in one variable.
Ex. P(x) = 4x7 – 3x2 + 5 Trinomial of degree 7
Q(y) = 5y2 + 9 Quadratic Binomial
R(s) = 75 Linear monomial
Note: In general, the polynomial in one variable x of degree 'n' is anxn + an – 1xn–1 + an–2xn–2 + ….+ a2x2 + a1x + a0, an
≠ 0, where an, an–1,… a2, a1, a0 all are constants.

40 Class-X MATHEMATICS P
W
Values and Zeroes of a Polynomial
If p(x) is a polynomial in variable x and a is any real number, then the value obtained by replacing x by a in p(x) is called
value of p(x) at x = a and is denoted by p(a).
For Example:
For a polynomial p(x) = 2x3 – 5x2 + 7 at x = 5
To, replace x by 5.
p(5) = 2(5)3 – 5(5)2 + 7 = 250 – 125 + 7 = 132
If for x = a, the value of the polynomial p(x) is 0 i.e., p(a) = 0; then x = a is a zero of the polynomial p(x).
For Example:
For polynomial p(x) = x – 2; p(2) = 2 – 2 = 0
∴ x = 2 or simply 2 is a zero of the polynomial p(x) = x – 2.
Note:
(i) A polynomial of degree n can have almost n zeroes which means if a polynomial is of degree 5, then it can have
at the most 5 zeroes.
(ii) Difference between zeroes and solutions
Let's try to draw a graph of the polynomial f(x) = x2 – 2x – 8
The following table gives the values of y or f(x) for various values of x.
x –4 –3 –2 0 4 5
y= x2 –2x –8 16 7 0 –8 0 7

Y
(–4, 16)
16
14
12
10
8
(–3, 7) (5, 7)
6
4
2
(–2, 0) (4, 0)
X'–10 –8 –6 –4 –2 O 2 4 6 8 10 X
–2
–4
–6
–8 (0, –8)
Y′

All the above values are the solutions of the polynomial f(x) = x2 – 2x – 8
From these values, –2 and 4 are the zeroes of the polynomial f(x) as it makes the value of f(x) equal to 0.

Polynomials 41
Geometrical Meaning of the Zeroes of a Polynomial
The graph of the polynomial is drawn on a graph sheet, it will cut or touches the x-axis in as many places as there are
zeroes of the polynomial. Thus, the zeroes of a polynomial f(x) are the x-coordinates of the points, where the graph of
y = f(x) intersects the x-axis.

y y y y

x' x x' x x' x x' x

y′ y′ y′ y′

Graph Showing One Zero Graph Showing Two Zeroes Graph Showing One Zero Graph Showing No Zero of
of f(x) of f(x) of f(x) f(x)

Graph of a Linear Polynomial


The graph of a linear polynomial f(x) = ax + b, a ≠ 0 is always a straight line. To draw the graph of a straight line we
need minimum two points, because a unique line will pass through two given points. The line represented by y = ax + b
 b 
crosses the X-axis at exactly one point, namely  − , 0  .
 a 
y
f(x)
(0, b)

x' O x
 b 
 − ,0
 a 
y′

Graph of a Quadratic Polynomial


The graph of any quadratic polynomial f(x) = ax2 + bx + c, a ≠ 0 is a parabola. It is basically a curved shape opening up
or down depending upon whether a > 0 or a < 0.
y
y = ax2 + bx + c

x' O x

y′

42 Class-X MATHEMATICS P
W
Case 1: Here, the graph cuts x-axis at two distinct points a and b. The x-coordinates of a and b are the two zeroes of
the quadratic polynomial ax2 + bx + c in this case (see Fig.).
y y

a b b a
x' O x x' O x

y′ y′
(i) (ii)

Case 2: Here, the graph cuts the x-axis at exactly one point, i.e., at two coincident points. So, the two points a and b of
Case (i) coincide here to become one point a (see Fig.).

y y

a
x' O x x' O a x

y′ y′
(i) (ii)

The x-coordinate of a is the only zero for the quadratic polynomial ax2 + bx + c in this case.
Case 3: Here, the graph is either completely above the x-axis or completely below the x-axis. So, it does not cut the
x-axis at any point (see Fig.).
y y

x' O x x' O x

y′ y′
(i) (ii)
So, the quadratic polynomial ax2 + bx + c has no zero in this case.

Polynomials 43
Graph of a Cubic Polynomial
Graphs of cubic polynomial does not have a fixed standard shape. The graphs of cubic polynomial will always cross
X-axis at least once and at most thrice.
y y

x' x x' x

y′ y′

Knowledge Hub
™ A non-zero constant is a polynomial of degree zero, but the degree of zero polynomial is not defined.
™ Zero of a polynomial is actually the solution of the curve or graph, y = f (x) and the line y = 0.
™ A polynomial of degree n ≥ 1 can have at the most n real zeroes.
™ A non-zero constant polynomial has no zeroes.
™ If the sum of the co-efficients of polynomial is zero, then (x – 1) is a factor of the polynomial.

EXAMPLE
1. Degree of polynomial 5 is ____________? 2. Find the value of the polynomial p(x) = 7x3 – 4x2
Sol. We know the degree of the polynomial is the – 8x + 3?
highest power of a variable in the polynomial. Sol. p(–1) = 7(–1)3 – 4(–1)2 – 8(–1) + 3
Let p(x) = 5 = 5 × 1 p(–1) = 7 × (–1) – 4×(1) – 8(–1) + 3
\ p(x) = 5x0 =–7–4+8+3
Therefore degree = 0 = 0.

Relationship Between Zeroes and Coefficients of a Quadratic Polynomial


Let α and β be the zeroes of a quadratic polynomial f(x) = ax2 + bx + c, a ≠ 0, then f(x) can be written in the form of
factors (x – α) and (x – β) using factor theorem as
f(x) = k(x – α)(x – β), where k is constant

⇒ ax2 + bx + c = k(x2 – βx – αx + αβ) ⇒ ax2 + bx + c = k(x2 –(α + β)x + αβ)
ax2 + bx + c = kx2 – k(α + β)x + kαβ

Comparing the coefficients of x2, x and constant terms on both sides, we get
a = k, b = –k(α + β) and c = kαβ
On dividing both b and c by a
b −k ( α + β ) c k αβ b c b c
=⇒ = and ⇒ = – (α + β) and = αβ ⇒ α + β = − and αβ =
a k a k a a a a

44 Class-X MATHEMATICS P
W
Hence,
b Coefficient of x
Sum of the zeroes =− =−
a Coefficient of x 2
c Constant term
Product of the zeroes= =
a Coefficient of x 2
Note:
If α and β are the zeroes of a quadratic polynomial f(x), then the polynomial f(x) is given by
f(x) = k{x2 – (α + β)x + αβ}
        or
f(x) = k{x2 – (Sum of the zeroes) x + Product of the zeroes} where k is any non-zero real number.

EXAMPLE
1. Find the zeroes of each of the following quadratic Let a, b be the zeroes of g(x)
polynomial and verify the relationship between −1 3
the zeroes and their coefficients: \a= and b =
3 2
(i) g(s) = 4s2 − 4s + 1 (ii) g(x)= 6x2 – 3 – 7x −1 3 7 − ( −7 )
\a+b= + = =
Sol. (i) Given, g(s) = 4s2 − 4s + 1 3 2 6 6
= 4s2 − 2s − 2s + 1 Coefficient of x
= −
= 2s (2s – 1) –1(2s – 1) Coefficient of x 2
⸫ g(s) = (2s – 1)(2s – 1) −1 3 −1 −3
and a . b = × = =
3 2 2 6
Value of g(s) is zero when
Constent term
2s – 1 = 0 or 2s – 1 = 0 =
Coefficient of x 2
1 1
i.e., s = and s= 2. For each of the following, find a quadratic
2 2
polynomial whose sum and product respectively
1 1
Product of zeroes = + =1 of the zeroes are as given. Also, find the zeroes of
2 2 these polynomials by factorization.
−(−4)
= (a + b) = 8 4
4 (i) − , , (ii) −2 3, − 9
3 3
− Coefficient of s Sol. (i) Let a, b be the zeroes of quadratic
=
Coefficient of s 2 polynomial
Constant term
Product of zeroes = a.b = −8 4
Coefficient of s 2 Given a + b = and a . b =
1 1 1 3 3
= . = We know
2 2 4
(ii) Given g(x) = 6x2 –7x –3 f(x) = k[x2 – (a + b) x + a . b]...(i)

= (3x + 1)(2x – 3)  8 4
k  x2 + x + 
 3 3
Value of g(x) is zero when 3x + 1 = 0
k
or 2x – 3 = 0 =
3
(
3x 2 + 8 x + 4 )
−1 3
k
i.e., x =
3
and x =
2 =
3
(
3x 2 + 6 x + 2 x + 4 )

Polynomials 45
k Squaring both side, we get
= 3 x ( x + 2 ) + 2 ( x + 2 ) 
3 ⇒ (α – β)2 = (1)2  ⇒ α2 – 2αβ + β2 = 1
k ⇒ {(α + β)2 – 2αβ} – 2αβ = 1
= ( 3 x + 2 )( x + 2 )
3 ⇒ (α + β)2 – 4αβ = 1

Now, the zeroes are given by f(x) = 0 ⇒ (3)2 – 4 × k = 1 ⇒ 4k = 8 ⇒ k = 2
2 −2 Therefore, the value of k is 2.
Thus, x = − and x = –2 i.e., a = and
3 3  1
b=–2 5. If both (x – 2) and  x −  are factors of
 2
(ii) We have, a + b = −2 3 and a . b = –9
px2 + 5x + r, then show that p = r.
So, the required quadratic polynomial will be
2 Sol. Let us suppose, f(x) = px2 + 5x + r
f(x) = k ( x + 2 3 x − 9) 
(from eqn. (i))
2
1
= k ( x + 3 3 x − 3 x − 9) Here, zeros α and β are 2 and respectively.
2
(
= k x + 3 3 x − 3 )( ) According to the question,
Constant term
Now, the zeroes are given by f(x) = 0 Product of zeros =
Coefficient of x 2
Thus, x = −3 3 and x =
3
r 1 r r
⇒ α × β = ⇒ 2× = ⇒ 1=
i.e., a = −3 3 and b = 3 p 2 p p

3. Find a quadratic polynomial whose zeroes are 4 ⇒ ⸫ r = p Hence proved.


and 3 respectively.
6. Find the zeroes of the quadratic polynomial
Sol. Let the quadratic polynomial be ax2 + bx + c and it’s 9x2 – 5 and verify the relation between the zeroes
zeroes be α and β and its coefficients.
Given, α + β = 4 + 3 = 7 Sol. We have, 9x2 – 5 = 0
and αβ = 4(3) = 12 2
or ( 3 x ) − ( 5) = 0
2
A quadratic polynomial whose sum and product of
its zeroes is given can be written as (3 x − 5)(3 x + 5) = 0 [a2 – b2 = (a + b)(a – b)]
f(x) = K [x2 – (sum of zeroes) x + product of z eroes]
⇒ (3 x − 5) = 0 and (3 x + 5) = 0
Where K is a real number 5 − 5
or x = and x =
⸫ Required quadratic polynomial 3 3
5 − 5
f(x) = K[x2 – 7x + 12] Therefore, the zeroes of 9x2 – 5 are and
3 3
4. If α and β are the roots (zeroes) of the polynomial
−coefficient of x
f(x) = x2 – 3x + k such that α – β = 1, find the value Sum of the zeroes =
of k. coefficient of x 2
5 5 −0
Sol. Since α and β are the roots (zeroes) of the ⇒ − = ⇒0=0
3 3 9
polynomial f(x) = x2 – 3x + k
constant term
Coefficient of x −(−3) Product of zeroes =
∴ α+β= − = =3 coefficient of x 2
Coefficient of x 2 1
 5  − 5  −5
Constant term ⇒  
  =
and αβ = =k  3  3  9
Coefficient of x 2
−5 −5
Given, α – β = 1 ⇒ = , Hence, relation verified.
9 9

46 Class-X MATHEMATICS P
W
7. If α and β are the zeroes of the quadratic 1 1
polynomial f(x) = ax2 + bx + c, then calculate the Since and are the zeroes of the
aα + b aβ + b
α2 β2
value of + . required polynomial.
β α

∴ sum of the zeroes
Sol. f(x) = ax2 + bx + c
1 1 aβ + b + aα + b
b c = + =
∴ α + β = − and αβ = aα + b aβ + b (aα + b)(aβ + b)
a a
We have, a (α + β) + 2b
2 2 3 3 =
α β α +β a αβ + ab(α + β) + b 2
2
+ =
β α αβ  −b 
3
a ×   + 2b
(α + β) − 3αβ(α + β)
=  a 
=  [using eq. (i)]
αβ c
   −b 
a 2 ×   + ab ×   + b 2
3 a  a 
 b  c  b 
 −  − 3   −  − ab + 2ba
 a  a  a 
= a ab b
c = 2 2 2 = 2 =
a c − ab + ab a c ac
a
a
−b3 bc Product of the zeroes
3
+3 2
= a a
 1  1  1
c =   = 2
a  aα + b   aβ + b  a αβ + ab(α + β) + b 2
α3 + β3 3abc − b3 1
⸫ = =
 [using eq. (i)]
αβ c  −b 
a 2c a 2 × + ab ×   + b 2
8. If α and β are the zeroes of the polynomial a  a 
ax2 + bx + c, find a polynomial whose zeroes are 1 1 1
= 2
1 1 2 2 = 2 =
and a c − ab + b a a c ac
aα + b aβ + b
a a
Sol. As α and β are the zeroes of the polynomial Hence, the required polynomial is given by
ax2 + bx + c x2 – (sum of zeroes) x + product of zeroes
−b c 1

∴α+β= and αβ =  ...(i) 2  b 
a a = x −   x +
 ac  ac

Polynomials 47
Summary

An algebraic expression f(x) of the form f(x) = a0 + a1x + a2x2 + ... anxn is a polynomial,
where a0, a1, ... an are real numbers and all the powers of x are non-negative integers.

U Degree of a polynomial is the highest exponent of the variable in the polynomial.


U Zero of a polynomial is a number for which the value of that polynomial is zero.
U Zeroes of a polynomial are points where graph of the polynomial intersect x-axis.

U f(x) = ax + b; where a, b are real and a ≠ 0, is a linear polynomial.


POLYNOMIALS

U The graph of a linear polynomial is a straight line.

U f(x) = ax2 + bx + c, where a, b, c are real, and a ≠ 0 is a quadratic polynomial.


U The graph of a quadratic polynomial is a parabola.
U Parabola opens upwards, if a is +ve.
U Parabola opens downwards, if a is –ve

U If f(x) = ax2 + bx + c is a quadratic polynomial, then:


(i) Sum of zeroes (α + β) = −Coefficient of x = − b
Coefficient of x 2 a
Constant term c
(ii) Product of=zeroes (α × β) =  [where α, β are the zeroes
Coefficient of x 2 a of the polynomial]
U Quadratic polynomial is x2 – (sum of zeroes)x + (product of zeroes)

48 Class-X MATHEMATICS P
W
NCERT Exercise

Sol. (i) x2 – 2x –8
Exercise-I
Using middle term splitting and factorizing, we
1. The graphs of y = p(x) are given in Fig. below, for get
some polynomials p(x). Find the number of zeroes ⇒ x2 – 4x + 2x – 8 = x(x – 4) + 2(x – 4) = 0

of p(x) in each case. = (x – 4)(x + 2) = 0
y y
⇒ x – 4 = 0 and x + 2 = 0 ⇒ x = + 4, and – 2
Hence, zeroes of polynomial equation
x2 – 2x – 8 are (4, –2)

x' O x x' O x Sum of zeroes = 4 – 2


(α + β) = 2
−(−2)
y′ y′ =
1
(i) (ii)
y y −(Coefficient of x)
=
(Coefficient of x 2 )
Product of zeroes = 4 × (–2)

x' O x x' O x (α × β) = –8
−(8)
=
1
y′ y′
(Constant term)
(iii) (iv) =
(Coefficient of x 2 )
y y
(ii) 4s2 – 4s + 1 = (2s – 1)2
The value of 4s2 – 4s + 1 is zero when 2s – 1
O x O x 1
x' x' = 0, i.e., s =
2
1
Therefore, the zeroes of 4s2 – 4s + 1 are and
y′ y′ 2
1
(v) (vi) 2
Sol. (i) 0 (ii) 1 (iii) 3 −(Coefficient of s )
Sum of zeroes =
(iv) 2 (v) 4 (vi) 3 (Coefficient of s 2 )
1 1 −(−4)
+ =
2 2 4
Exercise-2

⇒1=1
1. Find the zeroes of the following quadratic Constant term
Product of zerores =
polynomials and verify the relationship between Coefficient of s 2
the zeroes and the coefficients. 1 1 1
× =
(i) x2 – 2x – 8 (ii) 4s2 – 4s + 1 2 2 4
(iii) 6x2 – 3 – 7x (iv) 4u2 + 8u 1 1

⇒ =
(v) t2 – 15 (vi) 3x2 – x – 4 4 4

Polynomials 49
(iii) 6x2 – 3 – 7x = 6x2 – 7x – 3 4 
= (3x + 1)(2x – 3) 3x2 – x – 4 are  , −1
3 
The value of 6x2 – 3 – 7x is zero when 4 1  1
Sum of zeroes =   + (–1) =   = −  − 
3x + 1 = 0 and 2x – 3 = 0 3 3  3
−1 3 −(Coefficient of x)

⇒x= and x = =
3 2 (Coefficient of x 2 )
−1
Therefore, the zeroes of 6x2 – 3 – 7x are 4  −4 
3 Product of zeroes =   × ( −1) = 
3 3  3 
and
2 (Coefficient term)
−(Coefficient of x) =
Sum of zeroes = (Coefficient of x 2 )
(Coefficient of x 2 )
2. Find a quadratic polynomial each with the given
−1 3 − ( −7 ) 7 7 numbers as the sum and product of its zeroes

⇒ + = ⇒ =
3 2 6 6 6 respectively.
Constant term
Product of zeroes = 1 1
Coefficient of x 2 (i) , −1 (ii) 2, (iii) 0, 5
4 3
−1 3 −3 −1 −1 1 1

⇒ × = ⇒ = (iv) 1, 1 (v) − , (vi) 4, 1
3 2 6 2 2 4 4
Sol. (i) We know,
(iv) 4u(u + 2)
1
Therefore, zeroes of polynomial equation Sum of zeroes=α + β =
4
4u2 + 8u are 0, –2. Product of zeroes = αβ = –1
8 If sum and product of zeroes of any quadratic
Sum of zeroes = 0 + (–2) = –2 = −  
4 polynomial is given, then the quadratic polynomial
−(Coefficient of u ) equation can be written directly as:
=
(Coefficient of u 2 ) x2 – (α + β)x + αβ = 0
0
Product of zeroes = 0 × –2 = 0 = 1
4 x2 – x + (–1) = 0
(Coefficient term) 4
=
(Coefficient of u 2 ) 2
4x – x – 4 = 0
Thus, 4x2 – x – 4 is the required polynomial.
(v) t2 = 15 or t = ± 15
1
Therefore, zeroes of polynomial equation t2 – 15 (ii) 2,
3
are ( 15 , – 15 ) Let the polynomial be ax2 + bx + c, and its zeroes be
Sum of zeroes = 15 + (– 15 ) = 0 α and β
0 −(Coefficient of t ) 3 2 −b
= −   = α + β ==2 =
1 (Coefficient of t 2 ) 3 a
1 c
Product of zeroes = 15 × (– 15 ) αβ = =
3 a
15
= –15 = − If a = 3, then b = −3 2, c =1
1
2
(Coefficient term) Then the quadratic polynomial is 3 x − 3 2 x + 1 .
= (iii) Sum of zeroes = a + b = 0
(Coefficient of t 2 )
(vi) 3x2 – 4x + 3x – 4 = x(3x – 4) + 1(3x – 4) Product of zeroes = ab = 5
= (3x – 4)(x + 1) ∴ If a and b are zeroes of any quadratic polynomial,
then the quadratic polynomial equation can be written
Therefore, zeroes of polynomial equation directly as:-

50 Class-X MATHEMATICS P
W
x2 – (a + b)x + ab = 0 1 −b
α+β= − =
x2 – (0)x + 5 = 0 4 a
Thus, x2 + 5 is the quadratic polynomial. 1 c
α×β= =
(iv) Sum of zeroes = a + b = 1 4 a
Product of zeroes = ab = 1 if a = 4, then b = 1, c = 1
∴ If a and b are zeroes of any quadratic polynomial, Then the quadratic polynomial is 4x2 + x + 1.
then the quadratic polynomial equation can be written
(vi) Sum of zeroes = a + b = 4
directly as:-
Product of zeroes = ab = 1
x2 – (a + b)x + ab = 0
x2 – x + 1 = 0 ∴ If a and b are zeroes of any quadratic polynomial,
then the quadratic polynomial equation can be written
Thus, x2 – x + 1 is the quadratic polynomial.
directly as:-
1 1
(v) − , . x2 – (a + b)x + ab = 0
4 4
Let the polynomial be ax2 + bx + c and its zeroes be α x2 – 4x + 1 = 0
and β. Thus, x2 – 4x + 1 is the quadratic polynomial

5. Zeroes of quadratic polynomial x 2 + 6x + 9 are


Fill in the Blanks 2 and 7.
1. A polynomial of degree n has at the most _________
zeroes. Match the Following
2. Degree of remainder is always _________ than degree
of divisor. 1. Match the following with correct response.
3. A linear polynomial is represented by a _________ Column-I Column-II
4. The graph of quadratic polynomial opens upward when (Zeroes) (Quadratic
'a' is _________ . polynomials)
5. The zeroes of a polynomial p(x) are precisely the
x – coordinates of the points, where the graph of P. 3 and –3 (i) x2 + x – 42
y = p(x) intersects the _________ axis.
Q. 5 + 2 and 5 − 2 (ii) x2 – 9
True and False Statements
R. 1 (iii)  80 
1. Sum of zeroes of quadratic polynomial –9 and x2 +   x − 1
9  9 
(coefficient of x)
= −
(coefficient of x 2 ) S. –7 and 6 (iv) x2 – 10x + 21
2. Graph of a quadratic polynomial is a parabola.
3. The degree of the product of two polynomials each of (a) P-(i), Q-(iii), R-(ii), S-(iv)
degree 5 is always 5.
(b) P-(iii), Q-(ii), R-(iv), S-(i)
4. Product of zeroes of quadratic polynomial is
(c) P-(ii), Q-(iv), R-(iii), S-(i)
(constant term)
= − (d) P-(iv), Q-(i), R-(iii), S-(ii)
(coefficient of x 2 )

Polynomials 51
2. Match the following with correct response. (a) P-(i), Q-(ii), R-(iii), S-(iv)
(b) P-(iv), Q-(iii), R-(ii), S-(i)
Column-I Column-II
(Polynomial) (No. of zeros) (c) P-(iv), Q-(iii), R-(i), S-(ii)
(d) P-(iv), Q-(ii), R-(i), S-(iii)
P. y (i) 1

Assertion & Reason Type Questions


Direction (Q. 1-3): In the following questions, a statement
of Assertion (A) is followed by a statement of Reason (R).
Mark the correct choice as:
x
(a) Both Assertion (A) and Reason (R) are true
and Reason (R) is the correct explanation of
Assertion (A).
(b) Both Assertion (A) and Reason (R) are true but
Q. y (ii) 2
Reason (R) is not the correct explanation of
Assertion (A).
(c) Assertion (A) is true but Reason (R) is false.
(d) Assertion (A) is false but Reason (R) is true.
1. Assertion (A): The graph y = f(x) is shown in figure, for
x
the polynomial f(x). The number of zeroes of f(x) is 5.
y

R. y (iii) 3

x' O x

y′

Reason (R): The number of zero of the polynomial f(x)



x is the number of point of which f(x) cuts or touches the
y-axis.
2. Assertion (A): (2 − 3) is one zero of the quadratic
polynomial then other zero will be (2 + 3) .
S. y (iv) 0 Reason (R): Irrational zeroes (roots) may or may not
occur in pairs.
3. Assertion (A): The sum and product of the zeroes of
1 1
a quadratic polynomial are − and respectively.
4 4
x 1
x Then the quadratic polynomial is 2x2 + + .
2 2
Reason (R): The quadratic polynomial whose sum and

product of zeroes are given is x2 – (sum of zeroes)x +
product of zeroes.

52 Class-X MATHEMATICS P
W
1. If the zeroes of the quadratic polynomial ax2 + bx + c, c 8. The zeroes of the quadratic polynomial x2 + 99x – 127
≠ 0 are equal, then are
(a) c and a have opposite signs (a) Both positive
(b) c and b have same sign
(b) Both negative
(c) c and a have same sign
(c) One positive and one negative
(d) c and b have opposite signs
(d) Can't say
2. If one of the zeroes of the quadratic polynomial
(a – 1)x2 + ax + 1 is –3, then find the value of a 9. If one of the zeroes of a quadratic polynomial of the
form x2 + ax + b is the negative of the other, then it
4 4
(a) (b) − (a) Has no linear term and the constant term is
3 3
2 7 negative.
(c) (d) −
3 3 (b) Has no linear term and the constant term is
positive.
3. Number of zeroes of polynomial is equal to number of
points where the graph of polynomial (c) Can have a linear term but the constant term is
negative.
(a) Intersects x-axis
(b) Intersects y-axis (d) Can't say
(c) Intersects y-axis or x-axis 10. If (x + λ) is a factor of polynomial p(x) = x2 + 5x + 6, then
(d) None of the above the value of λ is

4. A polynomial of degree n has (a) 2


(a) At least n zeroes (b) 3
(b) Only one zero (c) Both (a) and (b)
(c) More than n zeroes (d) None of these
(d) Atmost n zeroes
11. If one of the zeroes of the cubic polynomial x3 + ax2 +
5. Which of the following does NOT represent a zero
polynomial? bx + c is –1, then the product of the other two zeroes
(a) p(x) = 0 (b) p(x) = x0 – 1 is
(c) p(x) = x0 (d) p(x) = 0.x0 (a) b – a + 1 (b) b – a – 1
6. If f(x) = 2x3
− 13x2+ 17x + 12, then find out the (c) a – b + 1 (d) a – b –1
remainder when f(x) is divided by (x + 2).
12. The zeroes of the quadratic polynomial x 2 + kx
(a) –90 (b) –85
+ k, k ≠ 0,
(c) –70 (d) 90
7. Find the remainder when x3 – ax2 + 6x – a is divided (a) cannot both be positive
by x – a. (b) cannot both be negative
(a) 2a (b) 5a (c) are always unequal
(c) 6a (d) 9a (d) are always equal

Polynomials 53
In order to input the right values into the AutoCAD
Very Short Answer Type Questions software, the engineer needs to calculate some basic
values.
1. Form the quadratic polynomial whose sum and product
of zeroes are 6 and 7. 1. Find the dimensions of the outer frame?
(a) (10 + 2x) m × (10 + 3x) m
2. Find the value of 'a' for which (x – 3) is a factor of f(x)
(b) (5 + 2x) m × (10x + 2) m
= x­2 – ax + 9.
(c) (10 + 2x) m × (5 + 2x) m
Short Answer Type Questions (d) None of these
2. A metal sheet of the minimum area is used to make
1. If α and β are the zeroes of the polynomial the frame. Find the minimum area of the metal sheet
ax2 + bx + c, then find the value of before cutting?
(i) α – β (ii) α2 + β2 (a) (4x2 + 30x + 50) m2
2. Find zero of the polynomial (b) (4x2 + 50x + 30) m2
2 s 2 - (1 + 2 2) s + 2
(c) (4x2 + 30x + 40) m2
(d) None of these
Long Answer Type Questions 3. What is the area of the required final metal frame?
(a) (3x2 + 40x) m2 (b) (4x2 + 30x) m2
1. If α and β are the zeroes of the polynomial x2 + 4x + (c) (2x2 + 30x) m2 (d) None of these
β
3, form the polynomial whose zeroes are 1 + and 4. If the area of the frame is 54 sq. m, what is the value
α α
1+ . of x?
β (a) 2.9 m (b) 1.5 m
(c) 2.5 m (d) None of these
Case Study Type Questions 5. Find the perimeter of the frame?
Case Study-I (a) 42 m (b) 36 m
(c) 48 m (d) None of these
RK Fabricators has got an
order for making a frame for a
client machine. For this, they Case Study-II
are using AutoCAD software While playing in garden, Shivansh saw a honeycomb
to create a constructible and asked her mother what is that. She replied that it's a
model that includes the honeycomb made by honey bees to store honey. Also,
relevant information such as she told him that the shape of the honeycomb formed
dimensions of the frame and is parabolic. The mathematical representation of the
materials needed. honeycomb structure is shown in the graph.
The frame will have a solid base and will be cut out of
a piece of steel. The final area of the frame should be
54 sq m. The diagram of the frame is shown below:
10 m

5m

54 Class-X MATHEMATICS P
W
y 2. The expression of the polynomial represented by the
40 graph is
30 (a) x2 – 49 (b) x2 – 64
20 (c) x2 – 36 (d) x2 – 81
10 3. Find the value of the polynomial represented by the
(–6, 0) (6, 0)
graph when x = 6.
x' –8 –6 –4 –2 O 2 4 6 8 x
–10
(a) – 2 (b) – 1
(c) 0 (d) 1
–20
–30 4. The sum of zeroes of the polynomial x2 + 2x – 3 is
(a) –1 (b) – 2
–40
y' (c) 2 (d) 1
1. Graph of a quadratic polynomial is in _________ shape. 5. If the sum of zeroes of polynomial at2 + 5t + 3a is equal
(a) straight line to their product, then find the value of a.
(b) parabolic (a) – 5 (b) – 3
(c) circular −5 −5
(c) (d)
(d) None of these −3 3

ANSWER KEY
Quick Recall
Fill in the Blanks
1. n 2. Less 3. Straight line 4. Positive 5. x

True and False Statements


1. True 2. True 3. False 4. False 5. False
Match the Following
1. (c) 2. (b)
Assertion & Reason Type Questions
1. (c) 2. (a) 3. (a)

Multiple Choice Questions


1. (c) 2. (a) 3. (a) 4. (d) 5. (c) 6. (a) 7. (b) 8. (c) 9. (a) 10. (c)
11. (a) 12. (a)

Subjective Questions
Case Study Type Questions
Case Study-I
1. (c) 2. (a) 3. (b) 4. (b) 5. (a)
Case Study-II
1. (b) 2. (c) 3. (c) 4. (b) 5. (d)

Polynomials 55
Competitive Level

Relationship Between Zeroes and Coefficients of a Cubic Polynomial


Let α, β, γ be the zeroes of a cubic polynomial f(x) = ax3 + bx2 + cx + d, a ≠ 0. Then, by factor theorem, ( x − α ) , ( x − β )
and ( x − γ ) are factors of f(x).
∴ f(x) = k ( x − α)( x − β)( x − γ )
⇒ ax3 + bx2 + cx + d = k ( x − α)( x − β)( x − γ )
2
⇒ ax3 + bx2 + cx + d = k{x3 – (α + β + γ ) x + (αβ + βγ + γα) x − αβγ }
2
⇒ ax3 + bx2 + cx + d = kx3 – k (α + β + γ ) x + k (αβ + βγ + γα) x − k αβγ
By comparing the coefficients of x3, x2, x and constant terms on both sides, we get
a = k, b = – k (α + β + γ ), c = (αβ + βγ + γα) and d = − k (αβγ )
On solving we will get the following results.
b Coefficient of x 2
Sum of the zeroes = a + b + g =− =−
a Coefficient of x3
c Coefficient of x
⇒ Sum of the products of the zeroes taken two at a time = ab + bg + ga= =
a Coefficient of x3
d Constant term
⇒ Product of the zeroes = abg = − = −
a Coefficient of x3
Note: Cubic polynomial having α, β, and γ as its zeroes is given by
f(x) = k{x3 – (α + β + γ)x2 + (αβ + βγ + γα)x – αβγ}, where k is any non-zero real number.

Division Algorithm for Polynomials


If f(x) and p(x) are any two polynomials such that p(x) ≠ 0, then we can find polynomials r(x) and q(x) such that
f(x) = p(x) × q(x) + r(x) i.e., Dividend = (Divisor × Quotient) + Remainder
where r(x) = 0 or degree of r(x) < degree of p(x).
(i) If r(x) = 0, p(x) is a factor of f(x).
(ii) If deg(f(x)) > deg(p(x)), then deg(q(x)) = deg(f(x)) – deg(p(x))
(iii) If deg(f(x)) = deg(p(x)), then deg (q(x)) = 0 and deg(r(x)) < deg(p(x))

Working Rule to Divide a Polynomial by Another Polynomial


Step 1: Arrange the terms of dividend and the divisor in the decreasing order of their degrees.
Step 2: T
 o obtain the first term of quotient, divide the highest degree term of the dividend by the highest degree term
of the divisor.
Step 3: T
 o obtain the second term of the quotient, divide the highest degree term of the new dividend obtained as
remainder by the highest degree term of the divisor.
Step 4: Continue this process till the degree of remainder is less than the degree of divisor.

56 Class-X MATHEMATICS P
W
EXAMPLE
1. Check whether first polynomial is a factor of As remainder must be zero
the second polynomial by applying the division (32 + a)x + (b – 48) = 0x + 0
algorithm.
∴ 32 + a = 0 ⇒ a = –32
x2 + 3x + 1, 3x4 + 5x3 – 7x2 + 2x + 2
And b – 48 = 0 ⇒ b = 48
Sol. We divide 3x4 + 5x3 – 7x2 + 2x + 2 by
∴ a + b = –32 + 48 = 16
x2 + 3x + 1
3. Find the value that must be subtracted from the
3x 2 − 4 x + 2
expression 8x4 + 14x3 – 2x2 + 8x – 12, so that it is
x 2 + 3 x + 1 3x 4 + 5x 3 − 7 x 2 + 2 x + 2
exactly divisible by 4x2 + 3x – 2.
3 x 4 + 9 x3 + 3 x 2 Sol. 4x2 + 3x – 2 8x4 + 14x3 – 2x2 + 8x – 12 2x2+2x–1
− − − 4 3
8x + 6x – 4x
2

– – +
− 4 x3 − 10 x 2 + 2 x + 2 3 2
8x + 2x + 8x – 12
− 4 x3 − 12 x 2 − 4 x 3 2
8x + 6x – 4x
+ + + – – +

2 x2 + 6 x + 2 –4x2 + 12x – 12
2
–4x – 3x + 2
2 x2 + 6 x + 2 + + –
− − − 15x – 14
0 ⸪ We get a remainder equal to 15x – 14. For exact
Since, here remainder is zero.
divisibility remainder must be equal to zero.
Hence, x2 + 3x + 1 is a factor of
∴ The expression that must be subtracted
3x4 + 5x3 – 7x2 + 2x + 2 = 15x – 14
Checking 3x4 + 5x3 – 7x2 + 2x + 2
4. If xn – pyn + qzn is exactly divisible by x2 – (ay + bz)x
= (3x2 – 4x + 2) (x2 + 3x + 1) + 0 p q
= 3x4 + 5x3 – 7x2 + 2x + 2 = Dividend + abyz, then find the value of n − n
a b
2. If the polynomial p(x) = 2x5 – 15x4 + 40x3 – 34x2 + 2
Sol. On factorizing x – ayx – bzx + abyz = x(x – ay)
ax + b is divisible by g(x) = x2 – 4x + 6. Then find – bz(x – ay)
the value of a + b.
= (x – ay)(x – bz)
Sol. We have,
Let, f(x) = xn – pyn + qzn is exactly divisible by (x – ay)
p(x) = 2x5 – 15x4 + 40x3 – 34x2 + ax + b and g(x) (x – bz)
= x2 – 4x + 6

∴ f(ay) = f(bz) = 0  (Using factor theorem)
As p(x) is divisible by g(x), the remainder will be

f(ay) = anyn – pyn + qzn = 0 ...(i)
zero.
Now, applying long division method,
f(bz) = bnzn – pyn + qzn = 0 ...(ii)
2 3 2
x – 4x + 6 2x5 – 15x 4 + 40x3 – 34x 2 + ax + b 2x – 7x + 8 Subtracting (ii) from (i)
anyn – bnzn = 0

5 4 3
2x – 8x + 12x
– + –
–7x4 + 28x3 – 34x2 + ax + b n bn z n
⇒ y =
...(iii)
–7x4 + 28x3 – 42x2

an
+ +
2
8x + ax + b Now substituting the value of yn in eq (ii), we get
 bn z n 
2
8x – 32x + 48
n n n
– + – b z
 n− p  + qz =0
(32 +a)x + (b – 48)  a 

Polynomials 57
Dividing all the terms of this expression by bnzn, we 1 a
have ⇒ = ...(i)
x −1 a+b+c
p q p q
1− + n = 0 ⇒ n − n =1 1 b
a n
b a b Similarly, = and ...(ii)
y −1 a + b + c
5. Let a, b, c, x, y and z be numbers such that a
1 c
b+c c+a a+b = ...(iii)
= ,b= ,c= . If xy + yz + zx z −1 a + b + c
x−2 y−2 z−2
Adding (i), (ii) and (iii)
= 67 and x + y + z = 2010, find the value of xyz + 5900. 1 1 1 a b c
+ + = + +
b+c c+a a+b x −1 y −1 z −1 a + b + c a + b + c a + b + c
=
Sol. Given, a = ,b = ,c
x−2 y−2 z−2 ( y − 1)( z − 1) + ( x − 1)( z − 1) + ( x − 1)( y − 1)
=1
b+c ( x − 1)( y − 1)( z − 1)
⇒x–2=
a ⇒ yz – y – z + 1 + xz – x – z + 1 + xy – y – x + 1
Adding 1 on both sides, we get = xyz – xy – zx – yz + x + y + z – 1
b+c ⇒ 2(xy + yz + zx) – 3(x + y + z) + 4 = xyz
⇒x–1= +1 ⇒ xyz = – 5892
a
a+b+c  [Using the values given in the question]
⇒x–1= xyz + 5900 = –5892 + 5900 = 8
a

Quartic Polynomial: If α, β, γ, δ are zeroes of a quartic equation ax4 + bx3 + cx2 + dx + e = 0, then
−b
α+β+γ+δ=
a
c
αβ + βγ + γδ + αγ + γδ + βδ =
a
−d
αβγ + αγδ + αβδ + βγδ =
a
e
αβγδ =
a

Highest Common Factor (HCF) or Geatest Common Factor (GCF)

H.C.F of given Polynomials


For the given two polynomials, f(x) and g(x), r(x) can be taken as the highest common factor, if
(i) r(x) is a common factor of f(x) and g(x) and
(ii) Every common factor of f(x) and g(x) is also a factor of r(x). The HCF, or highest common factor, is a frequent
abbreviation.
Highest Common Factor (H.C.F) of Polynomials by Division Method
‰ Let us consider two polynomials f(x), g(x).
f ( x)
‰ Divide the polynomials to get f(x) = g(x) × q(x) + r(x). Here the degree of g(x) is greater than the degree of r(x).
g ( x)
‰ If the remainder r(x) is zero, then g(x) is the H.C.F of polynomials.
‰ If the remainder ≠ zero, then again divide g(x) by r(x) to obtain g(x) = r(x) × q(x) + r1(x). Here if r(x) is zero then the
required H.C.F is r(x).
‰ If it is not zero, continue the process until we get the remainder equal to 0.

58 Class-X MATHEMATICS P
W
EXAMPLE
1. Find the H.C.F of x4 + 4x3 + x – 10 and x2 + 3x – 5 2. Find the H.C.F. of the polynomials x2 + 4x + 4 and
by using the division method ? x2 – 4.
Sol. Using division method, Sol. Factoring x2 + 4x + 4 by using the identities
x2 + 1x + 2
(a + b)2, we get (x)2 + 2(x)(2) + (2)2
x2 + 3x – 5 x4 + 4x3 + 0x2 + x – 10
= (x + 2)2
x4 + 3x3 – 5x2
– – + = (x + 2) (x + 2)
0+ x3 + 5x2 + x – 10 Also, factoring x2 – 4, we get
  x3 + 3x2 – 5x (x)2 – (2)2, by using the identities of a2 – b2.
– – +
0 + 2x2 + 6x – 10 = (x + 2) (x – 2)
2x2 + 6x – 10 Therefore, H.C.F. of x 2 + 4x + 4 and x 2 – 4 is
– – +
0 (x + 2).

L.C.M

L.C.M of the given Polynomials


The least Common Multiple (L.C.M) or the Lowest Common Multiple is the product of all the factors (taken once) of
the polynomials given with their highest exponents, respectively.
Method to calculate L.C.M of the given polynomials
Step 1: First, express each of the polynomials as a product of the powers of irreducible factors.
Step 2: Think about each irreducible component that appears (just once) in the given polynomials. Think about the
polynomial's highest exponent in its factored form for each of these factors.
Step 3: Now raise each irreducible factor to the greatest exponent and multiply them to get the L.C.M.

EXAMPLE
1. Find the lowest common multiple of 3x4 – 12x2 = 3x2 (x2 – 22), by using the formula

(2x2 - 4x), (3x4 - 12x2), a2 – b2 = (a + b) (a – b)



= 3x2(x + 2) (x – 2)
and (2x5 – 2x4 – 4x3)?
Third Polynomial = (2x5 – 2x4 – 4x3)
Sol. Given polynomials are
= 2x3(x2 – x – 2), by taking 2x3 common
(2x2 – 4x), (3x4 – 12x2),
= 2x3(x2 – 2x + x – 2)
and (2x5 – 2x4 – 4x3). = 2x3 (x + 1) (x – 2)
First polynomial = (2x2 – 4x) Therefore, the required L.C.M.
= 2x(x – 2), by taking 2x common = 12x3(x – 2)(x + 2)(x + 1).

Polynomials 59
Relationship Between HFC and L.C.M
Relation between the H.C.F, the L.C.M and the Product of the Polynomials
If f(x) and g(x) are two polynomials then we have the following relation, (H.C.F of f(x) and g(x)) × (L.C.M of f(x) and
g(x)) = f(x) × g(x)
Use the factorization method to determine the polynomials highest and lowest common factors. To determine the product
of two polynomial expressions, multiply them.

EXAMPLE
1. Find H.C.F & L.C.M of two polynomials 2x2 – x – 1 (4x2 + 8x + 3) (2x2 – x – 1)
and 4x2 + 8x + 3 and prove that the product of
= (2x + 1) (2x + 3) (x – 1) (2x + 1)
polynomials is the product of their L.C.M and
H.C.F? (2x + 1)2 (2x + 3) (x – 1)
Sol. Given two polynomials are 2x2 – x – 1 and 4x2 + = (x – 1) (2x + 1) (2x + 3) (2x + 1)
8x + 3. Hence proved.
By factoring 2x2 – x – 1, we get
2. Find the GCD pair of the following polynomials
= 2x2 – 2x + x – 1
= (x – 1) (2x + 1) (x3 + y3), (x4 + x2y2 + y4) whose LCM is
= (2x + 1) (x – 1) (x3 + y3) (x2 + xy + y2)
By factoring 4x2 + 8x + 3, we get Sol. p(x) = x3 + y3
= 4x2 + 6x + 2x + 3 = (x + y)(x2 – xy + y2)
= 2x (2x + 3) + 1(2x + 3) g(x) = x4 + x2y2 + y4 = [x2 + y2]2 – (xy)2

= (2x + 1)(2x + 3)
= (x2 + y2 + xy) (x2 + y2 – xy)
The common factor in both polynomials is
(2x + 1). L.C.M. = (x3 + y3) (x2 + xy + y2)
H.C.F = common factor = (2x + 1) (x + y) (x2 – xy + y2) (x2 + xy + y2)
L.C.M = remaining factors × common factor p( x) × g ( x)
G.C.D. =
= (2x + 1) (x – 1) (2x + 3) L.C.M.
= (2x + 3) (x – 1) (2x + 1) ( x + y )( x 2 − xy + y 2 ) × ( x 2 + y 2 + xy )( x 2 + y 2 − xy )
=
HCF and LCM of two polynomials are related by ( x + y )( x 2 − xy + y 2 )( x 2 + xy + y 2 )
the formula: Product of two polynomials = Product
of polynomials LCM and HCF. G.C.D. = x2 – xy + y2

60 Class-X MATHEMATICS P
W
Excluded from NCERT

(iii) Dividend = p(x) = x4 – 5x + 6


Exercise-I
= x4 + 0x2 – 5x + 6
1. Divide the polynomial p(x) by the polynomial g(x)
Divisor = g(x) = 2 – x2 = –x2 + 2
and find the quotient and remainder in each of the
following : –x2 –2
(i) p(x) = x3 – 3x2 + 5x – 3, g(x) = x2 – 2 –x2 + 2 x4 + 0x3 – 0x2 – 5x + 6
(ii) p(x) = x4 – 3x2 + 4x + 5, g(x) = x2 + 1 – x x4 – 0x3 – 2x2
(iii) p(x) = x4 – 5x + 6, g(x) = 2 – x2 – + +

Sol. (i) Given, Dividend = p(x) = x3 – 3x2 + 5x – 3 2x2 – 5x + 6


2x3 + 0x – 4
Divisor = g(x) = x2 – 2 – – +
x−3 –5x + 10
x − 2 x − 3x 2 + 5 x − 3
2 3
Therefore, after division we get,
Quotient = –x2 – 2
x3 + 0 x 2 − 2 x
Remainder = –5x + 10
– − +
2. Check whether the first polynomial is a factor
−3 x 2 + 7 x − 3
of the second polynomial by dividing the second
−3 x 2 + 0 x + 6 polynomial by the first polynomial:
+ – – (i) t2 – 3, 2t4 + 3t3 – 2t2 – 9t – 12
7x − 9 (ii) x2 + 3x + 1, 3x4 + 5x3 – 7x2 + 2x + 2
Therefore, after division we get, Quotient = x – (iii) x3 – 3x + 1, x5 – 4x3 + x2 + 3x + 1
3 and remainder = 7x – 9 Sol. (i) t2 – 3, 2t4 + 3t3 – 2t2 – 9t – 12
(ii) Given, Given, First polynomial = t2 – 3 = divisor
Dividend = p(x) = x4 – 3x2 + 4x +5 Second polynomial = 2t4 + 3t3 – 2t2 – 9t – 12
Divisor = g(x) = x2 + 1 – x = dividend
x2 +x–3 2t 2 + 3t + 4
x2 – x + 1 x4 + 0x3 – 3x2 + 4x + 5 t 2 − 3 2t 4 + 3t 3 − 2t 2 − 9t − 12
x4 – x3 + x2 2t 4 + 0t 3 − 6t 2
– + – − + +
x3 – 4x2 + 4x + 5
x3 – x2 + x 3t 3 + 4t 2 − 9t − 12
– + –
3
–3x2 + 3x + 5 3t + 0t 2 − 9t
–3x2 + 3x – 3 − − +
+ – +
8 4t 2 + 0t − 12
Therefore, after division we get, 4t 2 + 0t − 12
Quotient = x2 + x – 3 − − +
Remainder = 8 0

Polynomials 61
Since, remainder as 0. Therefore, t2 – 3 is a x2 + 2 x + 1
factor of 2t4 + 3t3 – 2t2 – 9t – 12. 3 x 2 − 5 3 x 4 + 6 x3 − 2 x 2 − 10 x − 5

(ii) 3x2 – 4x + 2 3x 4 −5 x 2
− +
x2 + 3x + 1 3x4 + 5x3 – 7x2 – 2x + 2
+ 6 x + 3 x 2 − 10 x −5
3

3x4 + 9x3 + 3x2


– – – 6 x3 − 10 x
− +
–4x3 – 10x2 + 2x + 2
–4x3 – 12x2 – 4x 3x 2 −5
+ + +
3x 2 −5
2x2 + 6x + 2 − +

2x2 + 6x + 2 0
– – –
Now since D = dq + r
0
Therefore, 3x4 + 6x 3 − 2x 2 − 10x – 5 = (3x 2 – 5)
Quotient = 3x2 – 4x + 2, Remainder = 0
(x2 + 2x + 1)
Since, the remainder is zero, x2 + 3x + 1 is a Now, on further factorizing (x2 + 2x + 1) we get,
factor of 3x4 + 5x3 – 7x2 + 2x + 2 x2 + 2x + 1 = x2 + x + x + 1
(iii) x2 – 1  [splitting the middle term]
x2 – 3x + 1 x5 – 4x3 + x2 + 3x + 1 = x(x + 1) + 1(x + 1)
x5 – 3x3 + x2 = (x + 1)(x + 1)
– + –
(x + 1)(x + 1) = 0
–x3 + 3x + 1
–x3 + 3x – 1 So, its zeroes are given by: x = −1 and x = −1.
+ – + Therefore, all four zeroes of given polynomial are:
2
5 5
Quotient = x2 – 1, Remainder = 2   , −   , −1 and −1.
3 3
Since, the remainder is not zero, x3 – 3x + 1 is
4. On dividing p(x) = x3 – 3x2 + x + 2 by a polynomial
a factor of x5 – 4x3 + x2 + 3x + 1 g(x), the quotient and remainder were x – 2 and –2x
3. Obtain all other zeroes of 3x4 + 6x3 – 2x2 – 10x – 5, + 4, respectively. Find g(x).
5 5 Sol. Given,
if two of its zeroes are and − .
3 3 Dividend, p(x) = x3 – 3x2 + x + 2
Sol. Since it is a quartic polynomial, it can have at most Quotient = x – 2
4 zeroes. Remainder = –2x + 4
5 5 As we know,
  and −   are zeroes of the polynomial f(x).
3 3 Dividend = Divisor × Quotient + Remainder
  5   5 2  5  3x − 5
2 ∴ x3 – 3x2 + x + 2 = g(x) × (x – 2) + (–2x + 4)
∴  x −    x +    =x −   =
  3    3   3 3 x3 – 3x2 + x + 2 – (–2x + 4) = g(x) × (x – 2)
 
 [(a – b)(a + b) = a2 – b2] Therefore, g(x) × (x – 2) = x3 – 3x2 + 3x – 2
(3x2 − 5) = 0 is a factor of given polynomial f(x). Now, for finding g(x) we will divide x3 – 3x2 + 3x
 [Factor theorem] – 2 from (x – 2)

62 Class-X MATHEMATICS P
W
x2 − x + 1 (iii) deg r(x) = 0
x − 2 x3 − 3x 2 + 3x − 2 The degree of remainder is 0 only when the
remainder left after division algorithm is a
x3 − 2 x 2
− + constant.
− x 2 + 3x − 2 p(x) = x2 + 1

− x2 + 2 x
g(x) = x
+ −

q(x) = x and r(x) = 1
x−2
Clearly, the degree of remainder here is 0.
x−2
− + Also, division algorithm is satisfied here.
0
Therefore, g(x) = (x2 – x + 1) Exercise-2
5. Give examples of polynomials p(x), g(x), q(x) and 1. Verify that the numbers given alongside of the cubic
r(x), which satisfy the division algorithm and polynomials below are their zeroes. Also verify the
(i) deg p(x) = deg q(x) relationship between the zeroes and the coefficients
in each case:
(ii) deg q(x) = deg r(x)
1
(iii) deg r(x) = 0 (i) 2x3 + x2 – 5x + 2; , 1, –2
2
Sol. (i) deg p(x) = deg q(x)
(ii) x3 – 4x2 + 5x – 2; 2, 1, 1
Degree of dividend is equal to degree of
Sol. (i) Given, p(x) = 2x3 + x2 – 5x + 2
quotient, only when the divisor is a constant
term. 1
And zeroes for p(x) are = , 1, –2
2
p(x) = 3x2 + 3x + 6

3 2
q(x) = x2 + x + 2
1 1 1 1
∴ p   = 2  +   − 5  + 2 = 0


g(x) = 3 and r(x) = 0 2 2 2 2
Quotient × Divisor + Remainder p(1) = 2(1)3 + (1)2 – 5(1) + 2 = 0

= g(x) q(x) + r(x) p(–2) = 2(–2)3 + (–2)2 –5(–2) + 2 = 0

= 3 (x2 + x + 2) + 0 1
Hence, , 1, –2 are the zeroes of
= 3x2 + 3x + 6 2
= Dividend 2x3 + x2 – 5x + 2.
= P(x) Now, comparing the given polynomial with
general expression, we get
Division algorithm satisfied.
ax3 + bx2 + cx + d = 2x3 + x2 – 5x + 2

As, you can see, the degree of quotient is equal
to the degree of dividend here.
⇒ a = 2, b = 1, c = –5 and d = 2
(ii) Let us take an example, p(x) = x2 + 3 is a Let α, β, γ are the zeroes of the cubic polynomial
polynomial to be divided by g(x) = x – 1. 1
ax3 + bx2 + cx + d, and we take a = , b = 1 and
So, x2 + 3 = (x – 1) × (x) + (x + 3) 2
g = –2, then
Hence, quotient q(x) = x
Coefficient of x 2
Also, remainder r(x) = x + 3 Sum of zeroes = −
Coefficient of x3
Thus, you can see, the degree of quotient −1
q(x) = 1, which is also equal to the degree of
⇒α+β+γ=
2
remainder r(x). 1 −1
⇒ + 1 + (–2) =
Hence, division algorithm is satisfied here. 2 2

Polynomials 63
3 −1 ab + bg + ga = 2 × 1 + 1 × 1 + 1× 2 = 5 =
⇒ −2 =
5 c
2 2 =
1 a
−1 −1 −(−2) − d

⇒ = Hence verified. abg = 2 × 1 × 1 = 2 = =
2 2 1 a
Coefficient of x Hence, the relationship between the zeroes and
Sum of product of zeroes =
Coefficient of x3 the coefficients are satisfied.
−5
⇒ αβ + βα + γα =
2 2. Find a cubic polynomial with the sum, sum of the
1   1 −5 product of its zeroes taken two at a time, and the
⇒  × 1 + (1 × −2) +  −2 ×  =

2   2 2 product of its zeroes as 2, –7, –14 respectively.
1 −5 1 −5 Sol. Let a, b and g be the zeroes of the required
⇒ − 2 −1 = ⇒ −3 =
2 2 2 2 polynomial, then
−5 −5 a+b+g=2
⇒ = Hence Verified.
2 2 ab + bg + ga = –7
Constant term
Product of zeroes = abg = –14
Coefficient of x3
( −2 ) \ Cubic polynomial can be written as
⇒ αβγ =
2 x3 – (a + b + g)x2 + (ab + bg + ga)x – abg = 0
1
⇒ × (1) × ( −2 ) = –1 x3 – 2x2 – 7x + 14 = 0
2
Hence, x3 – 2x2 – 7x + 14 = 0 is the required cubic

⇒ - 1 = - 1 Hence verified. polynomial.
Hence, the relationship between the zeroes and
3. If the zeroes of the polynomial x3 – 3x2 + x + 1 are
the coefficients are satisfied.
a – b, a, a + b, find a and b.
(ii) Given, p(x) = x3 – 4x2 + 5x – 2
Sol. Let p(x) = x3 – 3x2 + x + 1
And zeroes for p(x) are 2,1,1.
And zeroes are given as a – b, a, a + b
\ p(2) = 23 – 4(2)2 + 5(2)–2 = 0

p(1) = 13 – (4 × 12) + (5 × 1) – 2 = 0
Coefficient of x 2
Sum of zeroes = −
Hence proved, 2,1,1 are the zeroes of x3 – 4x2 + Coefficient of x3
5x – 2 − ( −3)
a–b+a+a+b=
Now, comparing the given polynomial with 1
general expression, we get; ⇒ 3a = 3
\ ax3 + bx2 + cx + d = x3 – 4x2 + 5x – 2
⇒a=1

a = 1, b = –4, c = 5 and d = –2 Thus, the zeroes are 1 – b, 1, 1 + b.
As we know, if a, b, g are the zeroes of the Constant term
cubic polynomial ax3 + bx2 + cx + d, then; Now, product of zeroes =
Coefficient of x3
−b 1

a+b+g= ⇒ (1 – b) × 1 × (1 + b) = −

a 2
c
ab + bg + ga =
⇒ (1 – b) (1 + b) = – 1
a
−d ⇒ (1 – b2) = – 1

abg = .
a ⇒ b2 = 2
Therefore, putting the values of zeroes of the
polynomial, ⇒b= 2
−(−4) −b Hence, 1 − 2 , 1, 1 + 2 are the zeroes of the
a+b+g=2+1+1=4= =
1 a polynomial x3 – 3x2 + x + 1.

64 Class-X MATHEMATICS P
W
4. If two zeroes of the polynomial x4 – 6x3 – 26x2 + Sol. By division algorithm,
138x – 35 are 2 ± 3 , find other zeroes. Dividend = Divisior × Quotient + Remainder
Sol. Since this is a polynomial equation of degree 4, Dividend – Remainder = Divisor × Quotient
hence there will be total 4 zeroes.
x 4 – 6x 3 + 16x 2 – 25x – 10 – x – a = x 4 – 6x 3 +
Let f(x) = x4 – 6x3 – 26x2 + 138x – 35
16x2 – 26x + 10 – a will be perfectly divisible by
Since 2 + 3 and 2 − 3 are zeroes of given x2 – 2x + k.
polynomial f(x).
let us divide x 4 – 6x 3 + 16x 2 – 26x + 10 – a by
∴ [x – ( 2 + 3 )] [x – ( 2 − 3 )] = 0 x2 – 2x + k
x2 – 4x + 1 = 0 
x2 − 4 x + (8 − k )
x2 – 4x + 1 is a factor of a given polynomial f(x).
x 2 − 2 x + k x 4 − 6 x3 + 16 x 2 − 26 x + 10 − a
Now we divide the given polynomial by x2 – 4x + 1.
x 4 − 2 x3 + kx 2
x 2 − 2 x − 35 − + −
x − 4 x + 1 x − 6 x − 26 x 2 + 138 x − 35
2 4 3
− 4 x3 + (16 − k ) x 2 − 26 x
4 3 2
x − 4x + x − 4 x3 + 8 x 2 − 4kx
− + −
+ − +
− 2 x3 − 27 x 2 + 138 x − 35
(8 − k ) x 2 − ( 26 − 4k ) x + 10 − a
−2 x3 + 8 x 2 − 2 x
+ − +
( 8 − k ) x 2 − (16 − 2k ) x + (8k − k 2 )
2
−35 x + 140 x − 35 − + −
2
− 35 x + 140 x − 35
+ − +
( −10 + 2k ) x + (10 − a − 8k + k 2 )
0
It can be observed that (–10 + 2k) = 0 and
So, x 4 – 6x 3 – 26x 2 + 138x – 35 = (x 2 – 4x + 1) (10 – a – 8k + k2) = 0
(x2 – 2x − 35)
For (–10 + 2k) = 0,
Now on further factorizing (x2 – 2x − 35), we get
2k = 10
02 – (7 − 5)x − 35 = x2 – 7x + 5x + 35 = 0
x(x − 7) + 5(x − 7) = 0 And thus, k = 5

(x + 5)(x − 7) = 0 For (10 – a – 8k + k2) = 0


So, its zeroes are given by: x = −5 and x = 7. 10 – a – 8 × 5 + 25 = 0
Hence, the other zeroes of given polynomial are: −5 10 – a – 40 + 25 = 0
and 7. –5–a=0
5. If the polynomial x4 – 6x3 + 16x2 – 25x – 10 is divided
Therefore, a = – 5
by another polynomial x2 – 2x + k, the remainder
comes out to be x + a, find k and a. Hence, k = 5 and a = –5

Polynomials 65
1. If the zeroes of the quadratic polynomial x 2 + (a) –67x – 77 (b) – 67x + 77
(a + 1) x + b are 2 and –3, then (c) 67x – 77 (d) 67x + 77
(a) a = –7, b = –1 (b) a = 5, b = –1 8. The value of
(c) a = 2, b = – 6 (d) a = 0, b = – 6
( x − y)2 ( y − z )2 ( z − x) 2
2. If the zeroes of the polynomial x3 – 6x2 – 45x + 162 + + is:
( y − z )( z − x) ( x − y )( z − x) ( x − y )( y − z )
are a – b, –a, a + b, then the value of a is
(a) 0 (b) 6
(a) 3 (b) –6
(c) 3 (d) –3
(c) 6 (d) 5
9. Find the degree of the polynomial (x + 1)(x­2 – x – x4 + 1).
3. Given that one of the zeroes of the cubic polynomial
(a) 2 (b) 3
ax3 + bx2 + cx + d is zero, the product of the other two
(c) 1 (d) 5
zeroes is
10. The degree of the polynomial, 5x4 – 9x2 + x9 is
c c
(a) – (b) (a) 2 (b) 4
a a
(c) 1 (d) 9
b 11. Find the value of p for which (x − 2) is a factor of
(c) 0 (d) –
a polynomial x4 − x3 +2x2 − px + 4.
(a) 10 (b) –9
4. For a given polynomial p(x) = x4 – 6x3 – 26x2 + 138x –
(c) 4 (d) –10
35, the sum of two zeroes is 4. Then the sum of other
two zeroes is: 12. When 6x­9 + 3x16 – p is divided by x + 1, the remainder
(a) 6 (b) 2 is 20. The value of p is
(c) 8 (d) 1 (a) –23 (b) –12
(c) 8 (d) 23
5. Find the sum of real values of y satisfying the
equation x2 + x2y2 + x2y4 = 525 and x + xy + xy2 13. If (x – k) is the H.C.F. of x2 + x – 12 and 2x2 – kx – 9,
= 35. then the value of k is
7 9 (a) –3 (b) 3
(a) (b)
2 2 (c) Both (a) and (b) (d) None of these
5 3
(c) (d) 14. If the polynomial x4 + x3 + 8x2 + ax + b is divisible by
2 2
x2 + 1, then the value of a + b is
6. Using factor theorem, if factorized : (a) 7 (b) 4
p(x) = 2x4 – 7x3 – 13x2 + 63x – 45, which of the

(c) 8 (d) 3
following is not a factor:
(a) (x + 1) (b) (x + 3) 15. f(x) = x4 – 2x3 + 3x2 – ax + b leaves remainder 5 and
(c) (x – 1) (d) (2x – 5) 19 on division by (x – 1) and (x + 1) respectively. If
f(x) divided by (x – 3), the remainder is
7. The value that must be added to f(x) = 3x4 + 13x3 –
29x2 + 100x – 77, so that the resulting polynomial is (a) 0 (b) 23
completely divisible by g(x) = x2 + 6x – 7. (c) 47 (d) –47

66 Class-X MATHEMATICS P
W
16. If the polynomial 3x­4 – 7x3 – 7x2 + 21x – 6 has one of is divisible by (x2 – 1) are
its zero as − 3 , then find the values of other zeroes. (a) –3, –3 (b) –3, 3
1 1 (c) 3, –3 (d) 4, 2
(a) 3, 2, (b) 3, ,−2
3 3 18. If f(x) = x + x 9 + x 25 + x 49 + x 81 is divided by
1 1 (x3 – x), then the remainder is
(c) − 3, 2, − (d) 2, 2, −
3 3 (a) x27 (b) x2 + 5x + 1
17. The value of p and q, such that x4 + px3 + 2x2 – 3x + q (c) 5x2 (d) 5x

Competitive Corner

1. If a and b are the zeroes of the polynomial P(x) = x2 7. If the zeros of polynomial f(x) = x3 – 3x2 + m are in
+ 3x + k such that a – b = 5, then the value of k is Arithmetic Progression, then the value of ‘m’ is
(a) – 4 (b) 5 (a) 3 (b) 2
(c) – 3 (d) 2 (c) 0 (d) –2
2. If a and b are the zeros of the quadratic polynomial 8. When 2x2 + 3x + 1 is divided by x + 2 then quotient
1 1
P(x) = x2 + qx – p then the value of + is and remainder are :
α β
(a) Q = 2x – 1, R = 3
p q
(a) (b) (b) Q = x + 1, R = 3
q p
(c) Q = 2x + 1, R = 3
−p −q
(c) (d) (d) None of these
q p
3. The polynomial, f(x) = (x – 1)2 + (x – 2)2 + (x – 3)2 + 9. The zeros of quadratic polynomial x2 + 7x + 10 will be:
(x – 4)2 has minimum value, when x = ........... (a) 2 and – 5 (b) – 2 and 5
(a) 40 (b) 20 (c) – 2 and –5 (d) None of these
(c) 10 (d) 2.5
10. Let a, b and c are the roots of the polynomial
4. If x + y + z = 0 & x ≠ 0, y ≠ 0, z ≠ 0 then the value of equation x 3 – 597x – 5236 = 0 then the value of
x2 y 2 z 2 (a3 + b3 + c3) is
+ + is?
yz xz xy (a) 597 (b) 15708
(a) 0 (b) 1 (c) 5236 (d) 10472
(c) 2 (d) 3
11. If a and b are the zeros of the polynomial f(x) = x2 – 5x
5. If any polynomial f(x) is divided by x2
– 9, then + k such that a – b = 1, then the value of k is
remainder is 3x + 2. If divided by it is (x – 3) the
(a) 6 (b) 2
remainder will be:
(c) 3 (d) 0
(a) – 7 (b) 7
(c) 11 (d) – 11 12. If any polynomial f(x) is divided by (x2 – 16) then
remainder is (5x + 3). If it is divided by (x + 4) then
6. When x100 – 2x51 + 1 is divided by x2 – 1, the remainder
is r(x). The value of r(–2) + r(2) is: the remainder will be:
(a) 0 (b) 4 (a) 17 (b) – 17
(c) 6 (d) 8 (c) 23 (d) – 23

Polynomials 67
1 1 18. If 3x2 − x3 + 5x − 2 is divided by x − 1 + x2, then the
13. If x + = 1 and y + = 1, then what is the value of remainder is
y z
(a) – 3 (b) 2
 1 
 z + + 1 . (c) 3 (d) − 2
 x 
(a) 0 (b) 1 19. If polynomials 3x3 + x2 – 4x + P and 2x3 + Px2 + 3x – 3
are divided by (x – 2), get the same remainder. What
(c) 2 (d) 3
will be the value of P.
14. If 3 p + 3 q + 3 r = 0 then the value of (p + q + r)3 is 1
(a) + 3 (b)
(a) 3pqr (b) 9pqr 3
(c) 27pqr (d) 0 1
(c) – (d) – 3
3
15. If (x + k) is a common factor of (x2 + px + q) and (x2 +
lx + m), then the value of k is: x y
20. + = −1 , (x, y ≠ 0), then the value of x3 – y3 is:
y x
(a) l + p (b) m – q
l−p m−q (a) 1 (b) – 1
(c) (d)
m−q l−p 1
(c) 0 (d)
16. If ax3 + bx + c is divisible by x2 + dx + 1, then: 2
(a) a2 + b2 = ac (b) a2 – c2 = ab 21. If the zeroes of the polynomial 64x3 – 144x2 + 92x – 15
(c) a2 – b2 = ac (d) a2 + c2 = ab are in A.P., then the difference between the largest and
the smallest zeroes of the polynomial is
1
17. If 3, – 1, – are zeros of cubic polynomial p(x), where 7
3 (a) 1 (b)
p(x) is 8
(a) 3x3 + 5x2 – 11x – 3 (b) 3x3 – 5x2 – 11x + 3 3 1
(c) (d)
(c) 3x3 – 5x2 – 11x – 3 (d) 3x3 + 5x2 + 11x + 3 4 2

ANSWER KEY

Multiple Choice Questions


1. (d) 2. (c) 3. (b) 4. (b) 5. (c) 6. (a) 7. (c) 8. (c) 9. (d) 10. (d)
11. (a) 12. (a) 13. (b) 14. (c) 15. (c) 16. (a) 17. (c) 18. (d)

Competitive Corner
1. (a) 2. (b) 3. (d) 4. (d) 5. (c) 6. (b) 7. (b) 8. (a) 9. (c) 10. (b)
11. (a) 12. (b) 13. (c) 14. (c) 15. (d) 16. (b) 17. (c) 18. (b) 19. (b) 20. (c)
21. (a)

68 Class-X MATHEMATICS P
W
Explanations

0 = 9a – 9 – 3a + 1
School Level 0 = 6a – 8
4
⇒a=
3
Quick Recall 3. (a) The number of zeroes is equal to the number of
times graph touches or intersect x-axis.
Assertion & Reason Type Questions
4. (d) Maximum number of zeroes of a polynomial
1. (c) Assertion (A) is true but Reason (R) is false. = Degree of the polynomial
As the number zero of polynomial f(x) is the number 5. (c) Here, p(x) = x0 or p(x) = 1 is a constant
of points at which f(x) cuts (intersects) the x -axis and
polynomial but not a zero polynomial as c ≠ 0.
number of zero in the given figure is 5. So A is correct
6. (a) If a polynomial f(x) is divided by (x – a) then
but R is incorrect. remainder will be equal to f(a)
2. (a) Both Assertion (A) and Reason (R) are true and f(x) = 2x3 −13x2 + 17x + 12
Reason (R) is the correct explanation of Assertion (A). f(−2) = 2(−2)3 − 13(−2)2 + 17(−2) + 12
As irrational roots/zeroes may or may not occur in = −16 − 52 − 34 + 12 = −90
pairs e.g., x 2 - x(2 - 3) = 0 7. (b) Given, p(x) = x3 – ax2 + 6x – a
3. (a) Both Assertion (A) and Reason (R) are true and When p(x) is divided by x – a, the value of f(a) will be
Reason (R) is the correct explanation of Assertion (A). the remainder
1 \ p(a) = a3 – a3 + 6a – a

Sum of zeroes = − and
4
p(a) = 5a
1 Thus, the required remainder is 5a
product of zeroes =
4 8. (c) In a quadratic equation, ax2 + bx + c, if a and c
2  1 1 are of opposite sign then the roots of the equation
Quadratic polynomial be x −  −  x +
 4 4 will always be of opposite sign.
2 1 1 1 x 1 Hence, both the zeroes of given polynomial
⇒ x +
x + ⇒  2 x2 + +  x2 + 99x – 127 are of opposite signs.
4 4 2 2 2
9. (a) If one of the zeroes (say α) of a quadratic
Quadratic polynomial be 4x2 + x + 1. So, both A and
polynomial x2 + ax + b is the negative of the other,
R are correct and R explains A. then sum of zeroes is:
α + (– α) = –a
Multiple Choice Questions ⇒ a = 0, which means polynomial has no linear term.
Product of zeroes is:
1. (c) Let m be the zero of the polynomial ax2 + bx + c = 0
α(– α) = b
b
\m+m= − ⇒ b = –α2
a
c c b has to be negative, which means constant term is
2
Also, m × m = ⇒ m = negative.
a a
2
Therefore polynomial has no linear term and the
m > 0 (Since square of any number cannot be negative) constant term is negative.
which is only possible when a and c have same sign.
10. (c) P(x) = x2 + 5x + 6
2. (a) As –3 is a zero of the given polynomial, so Since x + λ is a factor of p(x), by factor theorem
P(–3) = (a – 1)(–3)2 + a(–3) + 1
P(–λ) = 0

Polynomials 69
λ2 – 5λ + 6 = 0 2. If (x – 3) is a factor of f(x), then f(3) = 0
(λ – 3) (λ – 2) = 0 ⇒ f(3) = (3)2 – a(3) + 9

⇒ λ = 2, 3
⇒ 18 – 3a = 0
11. (a) Let p(x) = x3 + ax2 + bx + c
⇒a=6
Let a, b and g be the zeroes of the given cubic
polynomial p(x). Short Answer Type Questions
∴ a = –1 [Given] 1. As α and β are the zeroes of the polynomial
And p(–1) = 0 ax2 + bx + c.
⇒ (–1)3 + a(–1)2 + b(–1) + c = 0
b c
∴α+β= −
; αβ =

⇒ –1 + a – b + c = 0 a a
⇒ c = 1 – a + b...(i) (i) (α – β) = (α + β2 – 2αβ)
2 2

We know that, = (α2 + β2 + 2αβ – 4αβ)


− Constant term c = (α + β)2 – 4αβ
(−1)3 ,
Product of all zeroes = =

Coefficient of x3 1
2
abg = –c  b  4c
⇒ (–1)bg = –c   [∴ a = –1]
= −  −
 a a

⇒ bg = c
b 2 4c b 2 − 4ac
⇒ bg = 1 – a + b   [From equation (i)] = − =
a2 a a2
Hence product of the other two roots is 1 – a + b.
12. (a) Given, the quadratic polynomial is x2 + kx + k b 2 − 4ac

α–β=±
Sum of the roots = –k/1 = –k a
Product of the roots = k/1 = k (ii) α2 + β2 = α2 + β2 + 2αβ – 2αβ
If k is negative, = (α + β)2 – 2αβ
Sum of the roots is positive 2
 b c
Product of the roots is negative. =  −  − 2  
 a  a
So, one zero will be positive and other zero will be
negative b 2 − 2ac
=
If k is positive, a2
Sum of the roots is negative 2. Let 2s2 – (1 + 2 2 )s + 2 =0
Product of the roots is positive. On factoring,
So, both the zeros will be negative. 2s2 – s – 2 2 s + 2 =0
Therefore, the zeros of the quadratic polynomial in 2s2 – 2 2 s – s + 2 =0
both cases cannot be positive.
2s(s – 2 )–1(s– 2 ) = 0
Subjective Questions (2s – 1)(s – 2)=0
Now, 2s – 1 = 0
Very Short Answer Type Questions 2s = 1
1. Here, sum and product of zeroes are given. 1
s=
Sum of the zeroes = 6 2
Product of the zeroes = 7 Also, s – 2 = 0
Hence the polynomial formed
s= 2
= x2 – (sum of zeroes) x + Product of zeroes 1
Therefore, the zeros of the polynomial are and 2.
= x2 – 6x + 7 2

70 Class-X MATHEMATICS P
W
Long Answer Type Questions 4. (b) Area of frame = 54 m2
1. Since α and β are the zeroes of the polynomial 4x2 + 30x = 54
x2 + 4x + 3. 2x2 + 15x – 27 = 0
Then, α + β = –4, αβ = 3 2x2 + 18x – 3x – 27 = 0
For the required polynomial, Sum of the zeroes (x + 9)(2x – 3) = 0
β α
x = 1.5 or – 9
= 1 + +1+
α β 5. (a) Perimeter of frame = Perimeter of Outside
Rectangle
αβ + β2 + αβ + α 2 = 2(10 + 2x + 5 + 2x)
=
αβ
= 2(15 + 4x)
2 2
α + β + 2αβ = 2(15 + 4 × 1.5) = 42 m
=
αβ
Case Study-II
2 2
(α + β) (−4) 16 1. (b) Graph of a quadratic polynomial is a parabolic

= = =
αβ 3 3 in shape.
Product of the zeroes 2. (c) Since the graph of the polynomial cuts the x-axis
at (–6,0) and (6, 0). So, the zeroes of polynomial
 β  α  α β αβ
= 1 +  1 +  = 1 + + + are –6 and 6.
 α  β  β α αβ
Therefore, Required polynomial is p(x) = x2 – (– 6 +
2 2 2 2
α +β 2αβ + α + β 6)x + (– 6)(6) = x2 – 36
= 2 + =
αβ αβ 3. (c) We have, p(x) = x2 – 36
(α + β) 2 (−4) 2 16 Now, p(6) = 62 – 36 = 36 – 36 = 0
= = =
αβ 3 3 4. (b) Let, f (x) = x2 + 2x – 3. Then,
The required polynomial is x2 – (sum of zeroes) −Coefficient of x
Sum of zeroes =
x + product of zeroes Coefficient of x 2
−2
162 16  16 16  = = −2
or x − x + or k  x 2 − x +  1
3 3  3 3
5. (d) The given polynomial is at2 + 5t + 3a
Cased Study Type Questions Given, sum of zeroes = product of zeroes.
Case Study-I −5 3a −5
⇒ = ⇒a=
1. (c) Length = (10 + x + x) = (10 + 2x) a a 3
Breadth = (5 + x + x) = (5 + 2x) cm
2. (a) Length of steel plate, l = (10 + 2x)
Breadth of steel plate, b = (5 + 2x) Competitive Level
Area of steel plate, A = lb
= (10 + 2x)(5 + 2x) Multiple Choice Questions
= 50 + 10x +20x + 4x2
= 50 + 30x + 4x2 1. (d) Given, f(x) = x2 + (a + 1)x + b.
A = 4x2 + 30x + 50
And, 2 and –3 are zeroes of f(x)
3. (b) Area of frame to be cut = 10 × 5 = 50 m2
⇒ f(2) = 0
Area of frame left = 4x2 + 30x + 50 – 50 ⇒ (2)2 + (a + 1) × 2 + b = 0

= 4x2 + 30x m2
⇒ 4 + 2a + 2 + b = 0

Polynomials 71
⇒ 6 + 2a + b = 0 ...(i)
and f(–3) = 0 ⇒
(1 + y + y ) = 7
2 2

⇒ (–3)2 + (a + 1) × (–3) + b = 0
(1 + y + y ) 3
2 4


⇒ 9 – 3a – 3 + b = 0 1 + 2 y + 3 y 2 + 2 y3 + y 4 7
⇒ =
⇒ 6 – 3a + b = 0 ... (ii) 1 + y2 + y4 3
On subtracting Eq. (ii) from Eq. (i), we get  [using (a + b + c)2 =
5a = 0 ⇒ a = 0  a2 + b2 + c2 +
2ab + 2bc + 2ac]

⇒ 6 + b = 0 [from Eq. (i)] ⇒ 3 + 6y + 9y + 6y + 3y = 7 + 7y + 7y4
2 3 4 2

⇒ b = –6 ⇒ 4y4 – 6y3 – 2y2 – 6y + 4 = 0


⇒ 2y4 – 3y3 – y2 – 3y + 2 = 0 ...(iv)
Hence, a = 0 and b = –6
1
2. (c) Given, f(x) = x3 – 6x2 – 45x + 162 Using hit and trial method 2 and are the roots of
2
Roots are a – b, –a, a + b the above equation.
Hence from equation (iv)
−coefficient of x
sum of zeroes =  1
coefficient of x3 2y4 – 3y3 – y2 – 3y + 2 = (y – 2)  y −  (2y2 – 5y 
 2
( −6 )  + 2) = 0
⇒ a – b – a + a + b = −
 1
1 ⇒ (y – 2)  y −  (2y2 – 5y + 2) = 0

⇒a=6  2
3. (b) For the given polynomial, ax3 + bx2 + cx + d, 0 The term (2y2 – 5y + 2) has complex roots.
is the zero’s of this polynomial implies d = 0 1 5
Hence, the polynomial is x(ax2 + bx + c) ∴ sum of real values of y = + 2 =

2 2
The other two zero’s will be from ax2 + bx + c
c 6. (a) Factors of 45 = ±1, ± 3, ± 5, ± 9, ± 15, ± 45
Product of the other zero’s =
a If we put x = 1 in p(x)
4. (b) The given polynomial
p(1) = 2(1)4 – 7(1)3 – 13(1)2 + 63(1) – 45
p(x) = x4 – 6x3 – 26x2 + 138x – 35 is a quartic polynomial
p(1) = 2 – 7 – 13 + 63 – 45
ax4 + bx3 + cx2 + dx + e = 0 i.e., it has a degree 4. let = 65 – 65 = 0
a, b, g and d be its zeroes ∴ x = 1 or x – 1 is a factor of p(x).
We know that, If we put x = –1 in p(x)

α+β+ γ +δ =
−b
= −
( −6 ) p(–1) = 2(–1)4 – 7(–1)3 – 13(–1)2 + 63(–1) – 45
=6
a 1 = 2 + 7 – 13 – 63 – 45 = –112

\ Sum of other two zeroes = 6 – 4 = 2 \ x = –1 is not a factor of p(x).
5. (c) We have, x2 + x2y2 + x2y4 = 525 Similarly if we put x = 3 in p(x)
⇒ x2 (1 + y2 + y4) = 525  ...(i) p(3) = 2(3)4 – 7(3)3 – 13(3)2 + 63(3) – 45
p(3) = 162 – 189 – 117 + 189 – 45
And x + xy + xy2 = 35
= 162 – 162 = 0
⇒ x(1 + y + y2) = 35 ...(ii)
Hence, x = 3 or (x – 3) = 0 is the factor of p(x).
squaring the equation (ii) Now we shall factorise the given polynomial using is
x2(1 + y + y2)2 = 1225 ...(iii) that (x – 1) and (x – 3)
divide eq. (iii) by (i) p(x) = 2x4 – 7x3 – 13x2 + 63x – 45
∴ p(x) = 2x3 (x – 1) – 5x2 (x – 1) – 18x(x – 1)
( )
2
x2 1 + y + y 2 1225  + 45(x – 1)
=
x 2
(1 + y 2
+y 4
) 525 ⇒ p(x) = (x – 1) (2x3 – 5x2 – 18x + 45)

72 Class-X MATHEMATICS P
W
⇒ p(x) = (x – 1) [2x2 (x – 3) + x(x – 3) – 15(x – 3)]
( z − x )3
⇒ p(x) = (x – 1) (x – 3) (2x2 + x – 15)

R(x) = ...(iii)
( x − y )( y − z )( z − x)
⇒ p(x) = (x – 1) (x – 3) (2x2 + 6x – 5x – 15)

Adding (i), (ii) and (iii)

⇒ p(x) = (x – 1) (x – 3)[2x(x + 3) – 5(x + 3)]

⇒ p(x) = (x – 1) (x – 3) (x + 3) (2x – 5). ( x − y )3 ( y − z )3
= + +
7. (c) We know that, ( x − y )( y − z )( z − x) ( x − y )( y − z )( z − x)
f(x) = g(x) × q(x) + r(x) ( z − x )3
f(x) – r(x) = g(x) × q(x) ( x − y )( y − z )( z − x)
f(x) + {–r(x)} = g(x) × q(x)
LHS is completely divisible by g(x). ( x − y )3 + ( y − z )3 + ( z − x )3
=
...(iv)
Hence we can say, if we add –r(x) to f(x), then the ( x − y )( y − z )( z − x)
resulting polynomial is divisible by g(x). Special case: If a + b + c = 0, then a3 + b3 + c3 = 3abc
As x – y + y – z + z – x = 0
∴ (x – y)3 + (y – z)3 + (z – x)3 = 3(x – y)(y – z)(z – x)

3( x − y )( y − z )( z − x)
= =3
( x − y )( y − z )( z − x)
9. (d) Multiply the highest degree variable in first
132x bracket with the highest degree variable in second
bracket. Hence, the highest power we obtain is 5.

10. (d) Degree is the highest power of the variable in any



∴ r(x) = –67x + 77
polynomial. Therefore, degree of given polynomial
Therefore, we should add –r(x) = 67x – 77 to f(x), such is 9.
that resulting polynomial is divisible by g(x). 11. (a) p(x) = x4 − x3 + 2x2 − px + 4
8. (c) Let Since x − 2 is a factor of p(x), p(2) = 0
( x − y)2 So, p(2) = 24 − 23 + 2(2)2 − p(2) + 4 = 0

P(x) = ⇒ 16 − 8 + 8 − 2p + 4 = 0
( y − z )( z − x)
Multiplying numerator and denominator by (x – y), ⇒ 2p = 20
we get ⇒ p = 10
( x − y )3 12. (a) When 6x9 + 3x16 – p is divisible by x + 1, the

P(x) = ...(i) remainder is 20.
( x − y )( y − z )( z − x)
Let f(x) = 6x9 + 3x16 – p
( y − z )2 ⇒ f(–1) = 20
let Q(x) =
( x − y )( z − x) ⇒ 6(–1)9 + 3(–1)16 – p = 20
Multiplying numerator and denominator by (y – z), we ⇒ 6(–1) + 3 (1) – p = 20
get ⇒ –p = 23
( y − z )3 ⇒ p = –23

Q(x) = ...(ii)
( x − y )( y − z )( z − x) 13. (b) Let p(x) = x2 + x – 12
Since (x – k) is a factor of p(x) then using factor
( z − x) 2
Let R(x) = theorem
( x − y )( y − z ) P(k) = 0
multiplying numerator and denominator by (z – x), we ⇒ k­2 + k – 12 = 0
get ⇒ (k + 4)(k – 3) = 0

Polynomials 73
⇒ k = –4 = x4 –2x3 + 3x2 –ax + b we get
or k = 3 ...(i) f(x) = x4 –2x3 + 3x2 – 5x + 8

Let q(x) = 2x2 – kx – 9,
When f(x) is divided by (x – 3), then using factor
Since (x – k) is a factor of q(x), then using Factor
theorem q(k) = 0 theorem.
⇒ 2k2 – k2 – 9 = 0 Remainder = f(3)
⇒ k2 = 9 = (3)4 – 2(3)3 + 3(3)2 –5(3) + 8
⇒ k = ±3 ...(ii) = 81 – 2 × 27 + 3 × 9 – 15 + 8
From equation (i) and equation (ii), we get
= 81 – 54 + 27 – 15 + 8 = 47
k = 3 as the only common solution, therefore the value
of k will be 3. Hence remainder is 47 when f(x) = x4 – 2x3 + 3x2 –5x
14. (c) x4 + x3 + 8x2 + ax + b is divisible by x2 + 1 + 8 is divided by (x – 3).
∴ Remainder = 0 16. (a) Let f(x) = 3x­4 – 7x3 – 7x2 + 21x – 6
As one of its zeroes are − 3 and 3
∴ Using factor theorem ( x − 3) ( x + 3)

– – = x2 – 3 will be the factor of f(x) = 3x­4 – 7x3 – 7x2 +
7 21x – 6
– – Now, applying long division method

– –

x(a – 1) + (b – 7) = 0
⇒ a – 1 = 0, b – 7 = 0 7
⇒ a = 1, b = 7
So, a + b = 8

15. (c) f(1) = 5  [using factor theorem]
4 3 2
⇒ (1) – 2(1) + 3(1) – a(1) + b = 5 ⸫ f(x) = (x2–3) (3x2 –7x + 2)
⇒1–2+3–a+b=5 ( )( 2
)(
= x + 3 x − 3 3 x − 6 x − x + 2 )
= ( x + 3 )( x − 3 ) 3 x ( x − 2 ) − 1( x − 2 ) 
⇒2–a+b=5
⇒–a+b=3 ...(i)
And f(–1) = 19  [using factor theorem] = ( x + 3 )( x − 3 ) (3 x − 1)( x − 2)
4 3 2
⇒ (–1) – 2(–1) + 3(–1) – a(–1) + b = 19 1

∴ Other roots are 3, 2, and .
⇒ 1 + 2 + 3 + a + b = 19 3
⇒ a + b = 13  ...(ii) 17. (c) Given, x4 + px3 + 2x2 – 3x + q is divisible by
On adding equation (i) and (ii) x2 – 1 = (x – 1) (x + 1) [α2 – β2 = (α – β) (α + β]
2b = 16
\ (x – 1) and (x + 1) are the factors of given polynomial,
⇒b=8 i.e., f(1) = f(–1) = 0
On putting the value of b in equation (i)
f(1) = 1 + p + 2 – 3 + q = 0
–a + 8 = 3
⇒ p + q = 0 ...(i)
⇒ –a = –5
f(–1) = 1 – p + 2 + 3 + q = 0
⇒a=5
⇒ – p + q = –6 ...(ii)
Hence a = 5 and b = 8 By solving eqn (i) and (ii), we get
By putting the value of a and b in f(x)
p = 3 and q = –3

74 Class-X MATHEMATICS P
W
18. (d) We know that the degree of remainder is always ⇒ f(x) = 4x2 – 20x + 30

less than the degree of divisor. Hence, the remainder ⇒ f(x) = 4(x2 – 5x) + 30

will be a quadratic.
 25 
Let remainder ⇒ f(x) = 4  x 2 − 5 x +  + 30 – 25

 4
R(x) = ax2 + bx + c
2
 5
Given, f(x) = x + x9 + x25 + x49 + x81
⇒ f(x) = 4  x −  + 5
 2
2
f(x) = Q(x).(x3 – x) + R(x)
 5
Now, f(x) is minimum when  x −  is minimum
According to the question,  2
2

f(x) = Q(x).x(x + 1)(x – 1) + R(x)  5 5
i.e. when  x −  = 0 ; when x = = 2.5
So, f(0) = R(0)  2 2
= c Alternatively,
= 0 f(x) = (x – a1)2 + (x – a2)2 + ... + (x – an)2


f(1) = R(1) a1 + a2 + ... + an
Assumes minimum value at
= a + b + c n
5 = a + b + c f(x) = (x – l)2 + (x – 2)2 + (x – 3)2 + (x – 4)2


⇒ a + b = 5 ...(i) 1 + 2 + 3 + 4 10
x
⇒ minimum value at x = = = = 2.5

f(–1) = R(–1) 4 4

–5 ⇒ a – b + c 4. (d) If x + y + z = 0, then x + y + z = 3x . y . z
3 3 3


⇒ a – b = –5 ...(ii) x2 y 2 z 2

= + +
By solving eqn (i) and (ii), we get yz zx xy

a = 0, b = 5
x3 y3 z3
\ R(x) = ax2 + bx + c
= + +
xyz zxy xyz
= ax2 + 5x + 0 = 5x
x3 + y 3 + z 3
=
Competitive Corner x ⋅ y ⋅ z
3x ⋅ y ⋅ z
1. (a) Given a – b = 5 ...(i) = = 3
as a + b = –3 ...(ii) x ⋅ y ⋅ z
Add (i) & (ii) 5. (c) Let f(x) = (x2 – 9) Q(x) + 3x + 2

a = 1, b = –4 where Q(x) is the quotient

ab = k = –4
f(3) = 3(3) + 2 = 11
2. (b) p(x) = x2 + qx – p 6. (b) Let f(x) = x100 – 2x51 + 1, and f(x) = g(x)(x2 – 1)

a + b = –q, ab = –p + ax + b be the division

1 1 α + β −q q Then, 0 = f(l) = g(1)(12 – 1) + a(1) + b = a + b, and 4


+= = = = f(–l) = g(–1)((–1)2 – 1) + a(–l) + b = –a + b.
α β
αβ −p p
Hence a = –2, b = 2.
3. (d) The given equation is f(x) = (x – l)2 + (x – 2)2
+ (x – 3)2 + (x – 4)2 Thus the remainder is –2x + 2.

f(x) = (x2 + 1 – 2x) + (x2 + 4 – 4x) + (x2 + 9 – 6x) +



r(2) = –2
(x2 + 16 – 8x)
r(–2) = 6
⇒ f(x) = 4x2 – 2 (1 + 2 + 3 + 4)x +30

r(2) + r(–2) = 4

Polynomials 75
7. (b) f(x) = x3 – 3x2 + m 25 – 1 = 4ab
Let zeros be a – b, a, a + b as zeros are in A.P. 24
= = 6
αβ
4
a− b +a+a+ b =3

∴ value of k = ab = 6.
3a = 3
12. (b) Let the quotient be q(x)

a=1
f(x) = (x2 – 16)q(x) + (5x + 3)
(1 – b)1 + (1 + b)1 + (1 – b) (1 + b) = 0
f ( x) ( x + 4)( x − 4) 5x + 3
3 – b2 = 0 = q( x) +

( x + 4) x+4 x+4
⇒ b= ± 3
Remainder = Remainder when 5x + 3 is divided by

\ Product of root = –m x+4
(1 − 3) ⋅ (1 + 3)(1) =−m = 5(–4) + 3 = –17

1 1

⇒ 1 – 3 = –m 13. (c) x + = 1 and y + =1
y z

⇒m=2
Calculation
8. (a) From the division, we observe that the quotient
1
is 2x – 1 and the remainder is 3. x+ =1
y
Quotient 1 y −1
⇒ x = 1− ⇒ x =
y y
2x – 1
1 y
x+2 2x2 + 3x + 1
⇒ = ...(i)
2
x y −1
2x + 4x
(–) (–) Also,
–x + 1 1
y+ =1
–x – 2 z
(+) (+) Remainder
1 1
3
⇒ =1–y ⇒ z=
z 1− y
Hence, the quotient is 2x – 1 and the remainder is 3. −1

⇒z= ...(ii)
9. (c) x2 + 7x +10 = x2 + 2x + 5x + 10 = 0 y −1
x(x + 2) + 5 (x + 2) = 0 Adding (i) and (ii), we get
(x + 2) (x + 5) = 0 1  y   −1 
z=
+  + 
x + 2 = 0 and x + 5 = 0 x  y − 1   y − 1 
x = –2 and x = –5 1 y −1
⇒z+ = 1 1
The zeroes of the polynomial x2 + 7x +10 are –2 and –5. ⇒z+ = 1 ⇒ z + +1 = 2
x y −1 x x
10. (b) x3 – 597x – 5236 = 0 14. (c) p1/3 + q1/3 + r1/3 = 0 ... (i)
Sum of roots = a + b + c = 0 If a + b + c = 0 then
Also, abc = 5236 a3 + b3 + c3 = 3abc
Since a + b + c = 0 ∴ by equation (i)
⇒ a3 + b3 + c3 = 3abc (p1/3)3 + (q1/3)3 + (r1/3)3 = 3 × p1/3 × q1/3 × r1/3
= 15708 ⇒ p + q + r = 3(pqr)1/3
11. (a) f(x) = x2 – 5x + k ∴ (p + q + r)3 = 27 pqr
& a – b = 1 15. (d) Let x2 + px + q = 0 ....(i)
& a + b = 5 & ab = k 2
and x + lx + m = 0 ....(ii)
we know (a + b)2 – (a – b)2 = 4ab If both equations have the same factor (x + k), then
subtracting equation (i) from (ii), we get

76 Class-X MATHEMATICS P
W
(p – l)x + (q – m) = 0 19. (b) Let P(x) = 3x3 + x2 – 4x + P
Common factor is (x + k) & Q(x) = 2x3 + Px2 + 3x – 3
Therefore putting x = –k in above equation, we get,

P(2) = Q(2) (Given)
(p – l)(–k) + (q – m) = 0
24 + 4 – 8 + P = 16 + 4P + 6 – 3
m−q
⇒k=
l−p
⇒ 20 + P = 19 + 4P
16. (b) 1 = 3P
ax – ad
1
x2 + dx + 1 ax3 + bx + c =P
3

ax3 + adx2 + ax
– – – x y
20. (c) If + =−1
– adx2
– ax + bx + c y x
– adx – ad2x – ad
2
simplifies to x2 + xy + y2 = 0
+ + +
(ad2 – a + b)x + (c + ad) as (x3 – y3) = (x – y)(x2 + xy + y2) = 0 using above result
21. (a) Let (a + b), a, (a – b) be the three zeroes of the
 ax3 + bx + c is divisible by x2 + dx + 1

given polynomial

∴ remainder should be zero
64x3 – 144x2 + 92x – 15
∴ (ad2 – a + b)x + (c + ad) = 0 × x + 0

⇒ ad2 – a + b = 0 ...(i) −(−144) 9
∴ (α + β) + α +=
(α − β) =
and c + ad = 0 ⇒ d = –c/a 64 4
by equation (i) 9
⇒ 3α =
 −c 
2
4
a  − a + b =0
 a  3
2
⇒ α=
c 4
– a + b = 0
a −(−15)
Also, (α + β)α(α − β) =
⇒ c2 – a2 + ab = 0
64
⇒ a2 – c2 = ab
3 3 3  15
⇒  + β  − β =
17. (c) x3 −  3 − 1 − 1  x 2 + 3(−1) + (−1)  −1  + (3)  −1   x  4 4 4  64
 3   3   3 
9  5
 −1  ⇒  − β2  =
2
−(3)(−1)   =
 3 
0  16  16
5 x 11 x
⇒ x3 − − −1 = 0 1
3 3 ⇒ β2 =
4
⇒ 3 x3 − 5 x 2 − 11x − 3 =
0 1
∴ β=±
18. (b) –x + 4 2
x2 +x–1 –x3 + 3x2 + 5x – 2 3 1
When α = and β =
–x3 – x2 + x 4 2
+ + –
4x2 + 4x – 2 5 3 1
So, the zeroes will be , ,
4x2 + 4x – 4 4 4 4
– – +
5 1
2 Hence, required difference = − =1
4 4

Polynomials 77

You might also like